Cs Unit 2 - Time Domains

You might also like

Download as docx, pdf, or txt
Download as docx, pdf, or txt
You are on page 1of 66

1

CONTROL SYSTEM (EEE1008) FOR B. TECH EE & EEE (CORE)/


CONTROL SYSTEM (EEE1408) FOR B. TECH ETC (CORE)

UNIT – 2
(MODULE- II)

(MODULE- II)

VOL-2: K 1
1. TOTAL RESPONSE COMPONENTS
2. SOME DEFINATIONS
3. TIME DOMAIN PERFORMANCE OF CONTROL SYSTEM
4. STANDARD TEST SIGNAL
5. TIME RESPONSE OF 1ST ORDER SYSTEM
6. TIME RESPONSE OF SECOND ORDER SYSTEM
7. STEADY STATE ERRORS AND ERROR CONSTANTS

VOL-2: K 2

1. TIME RESPONSE OF POSITIONAL SERVOMECHANISM

VOL-2: K 3
1. CONCEPT OF STABILITY
2. NECESSARY CONDITION OF STABILITY:
(a) HURWITZ STABILITY CRITERION
(b) ROUTH’S STABILITY CRITERION
(c) ROUTH – HURWITZ CRITERION
(d) Example’s 1 to 8

VOL-2: K 4
1. ROOT LOCUS.
2. OTHER PROPERTIES OF ROOT LOCUS.
3. ROOT CONTOURS
4. SYSTEM WITH TRANSPORTATION LAG
5. SENSITIVITY OF THE ROOTS OF THE CH. EQN.:
2

VOL – 2
MODULE – II
Time Domain Analysis:

Total Response; standard test signals, Time response of first order systems, Characteristic Equation of
feedback control systems, Transient response of second order systems, time domain specifications, Steady
state response, Steady state errors and static error constants. Generalized error series and generalized
error coefficients. The concept of stability: Hurwitz stability criterion. Routh stability criterion, relative
stability by shifting origin in s-plain; Root Locus technique: the root locus concept, construction of root loci,
effect of adding poles and zeros to G(s) H(s) on the root locus.
K1:
1. Total Response Components

A. Total or Complete Response = Zero – state response (zero initial conditions) + zero –input response (initial
conditions present)
i. Zero – state response = C(s) or C(t) ; Taking all initial conditional (or states) zero
zero state ¿ L−1 [G ( s ) R( s)] ¿
ii. Zero – input response = C(s)

¿
Zero input
Ch. Polynomical
× Initial values∨C ( t )=L
−1 1
Ch. Poly [ ]
× Initial condition ; No input or zero

input
Thus C(s) = C(s) + C(s) or C(t) = C(t) + C(t)
(Total) (Zero state) (Zero input) (Total) (Zero state) (Zero input)

B. Alternative to A:
The total response= Natural Response + Forced Response.
(i) The natural component consists of all the characteristic root terms in the partial fraction expression i.e. the
denominator polynomial of G(s) i.e. the response due to all poles excepting the poles corresponding to the input.
(ii) The forced response component is the remainder (i.e. the rest leaving the natural component) of the complete
response (i.e. the after deducting the natural response) and is composed of the terms associated with the input
term; I. e. the response due to the pole corresponding to the input only.

C. Another way:
Total response = Transient Response + Steady State Response
(i) Transient Response = the exponential natural response term decays with time.
(ii) Steady State Response = the constant force (or steady state) term i.e. remained, when the transient response
vanishes for t-> ∞
d2 y dy −dr ( t ) −3 t
Example: A system described by +6 +8 y = +5 r ( t )∧r ( t )=7 e
dt 2
dt dt
y ( o ) =0 , y ' ( o )=1 ,r ( o )=¿
7
Laplace transform of the given equation is:
s2 Y ( s ) −sy ( o )− y ' ( o )+ 6 [ sY ( s )− y ( o ) ]+ 8 Y ( s ) =−[ sR ( s )−r ( o ) ] +5 R(s)
Or ( s2 +6 s+8 ) Ys=[ sy ( o ) + y ' ( o )+ 6 y ( o ) +r ( o ) ] + R ( s ) [−s+ 5 ) ¿ ⋯ ⋯ ⋯ ⋯ ⋯ ⋯ ⋯ ⋯ (a)
A: (1) Assuming initial conditions to be zero,
Y ( s) −s +5 −3 t 7
G ( s )= = 2 ; if r ( t )=7 e , R ( s )=
R( s) s +6 s+8 (s+3)
Hence Y(s) = System output in Laplace domain = G(s) R(s) =7(-s+5) / (s 2+6s+8) (s+3) =Y(s) (zero state)
I. e obtained from Transfer function taking initial (state)-conditions to be zero.
3

(2) If we do not neglect initial conditions (state) i.e. if we consider the initial conditions, then another component of
Y(s) is present which obtained as:

Y ( s ) [ s +6 s+ 8 ]=[ sy ( o ) + y ( o ) +6 y ( o )+ r ( o ) ] + R ( s ) [−s+5 ) ¿ ⋯ ⋯ ⋯ ⋯ ⋯ ⋯ ⋯ ⋯(a)


2 '

[ ] [ ]
' (o )
sy ( o ) + y + 6 y ( o ) +r ( o ) R ( s ) (−s +5 )
Hence Y ( s )= ( zeroinput ) + ( zero state )
( s +6 s +8 )
2
( s2 +6 s+ 8 )
(Total)
0+ 1+7 8 7(−s+5)
Hence Y(s) = = ∧Y ( s )=
( s +6 s +8 ) ( s+2 ) ( s+ 4)
2
( s+ 2 )( s+ 4 ) ( s+ 3)
(Zero input) = (zero state)
AZ
B: Alternatively:
Y(s) = Y(s) (Force component) +Y(s) (natural component) −dr ( t )
L( +5 r ( t ) ¿
(Total) dt

(Initial Condition is also considered): ¿


[ 7 (−s+ 5 )
( s +6 s+8 ) ( s+3 )
2 ] (zero state)+
[( 8
s +6 s +8 )
2
] ( zero input )

7 (−s+5 )+ 8( s+3) a b c
¿ = + +
( s+2)(s +4 )(s +3) s+3 s +2 s+4

s +59 56
Wherea= /s=−3 ¿ =−56
( s+ 2 )( s+ 4 ) −1× 1

s +59 57
b= / s=−2= =28.5
( s+ 3 )( s+ 4 ) 1× 2

s +59 55
c= /s=−4= =27.5
( s+3 ) ( s +2 ) −1 ×−2

Forced response component


28.5 27.5 Natural response component
Hence Y(s) = ( ) −56 (I. e. Associated with pole of input
s +3 only)
+ +
s+ 2 s+ 4 ( )
(I.e. from the poles of system G(s))

(Total)

y ( t ) ( Foced )=L
−1
( −56
s +3 )
=−56 e
−3 t
; y ( t ) ( Natural )=L
−1
( 28.5
s +2 s+ 4 )
+
27.5
=28∙ 5 e
−2 t
+27 ∙ 5 e
−4 t

2. SOME DEFINATIONS
(1) Transfer Functions: The transfer function of a linear time - invariant system is defined to be the ratio of
the Laplace transform of the output variable to the Laplace transform of the input variable under the
assumption that all initial conditions are zero.

(2) Order of the system: The highest power of the complex variable‘s’ in the denominator of the transfer
function determines the order of the systems.

(3) Types of Feedback Control System.


The open loop transfer function (G(s)) of a unity feedback system can be written in two standard forms:-
4

(i) The time - constant form and (ii) The pole zero form.
K ' ( T z 1 s+1 ) ( T z 2 s+1 )
(i) G ( s )= n ------------- (Time constant form) ------ (1)
s ( T p 1 s +1 ) ( T p 2 s +1 )
K ' ( s + z 1 )( s+ z 1 )
(ii) G(s)= n -------------------------(Pole zero form) – (2)
s ( s+ p1 ) ( s + p2 ) R(s)
G(s)
-
The gains in the two forms are related by B(s)
πi zi
K= K ' ----------- (3)
π pi
Equ ation (1) involves the term s n in the denominator which corresponds to the number
integrations in the system.
Control systems are classified in accordance with the number of integration in the open-loop transfer
function G(s)
E. g n =0 Type – 0 system
n =1  Type – 1 system
n=2  Type 2 system
n=3  Type 3 system.
5

n=n  Type –n system


Time Domain Performance of Control System:
Control systems inherently are time domain system for which the time response becomes an
important factor for design and analysis of systems. If the variables of a system are expressed as function
of time, the analysis or design proceeds in the time domain.
Systems response is the effect of the system for which the cause is system input. Even if input is
steady the response is not steady for some initial time. It becomes steady only after some time has
elapsed. The response of any system can thus be considered as the same of the transient response, C t (t)
and the steady state response, Cs s (t).
I.e. C (t) = C t (t) + Cs s (t)
Also Lim C t (t) = 0
t∞
Lim C (t) = Cs s (t)
t∞
Typical Test Input Signals for the Time Domain Analysis of Feedback Control System.
In, practice, the input excitation to a feedback control system is not known ahead of time. In most cases,
the actual inputs vary are random fashions with respect to time. For instance, in radar tracking system the
position and speed of the target to be tracked may vary in any unpredictable manner. So they cannot be
expressed mathematically by any simple equation. However, for the purpose of analysis and design, it is
necessary to assume some types of input functional so that the performance of a system can be analyzed
with at least these test signals. In a design problem, performance criteria are derived with respect to these
test signals, and linear systems are designed to meet the criteria.
3. Time domain specifications: The commonly used specifications are:
(a) Steady State Error: Due to inertia and friction in mechanical system, capacitance and resistance in
electrical systems, there is always some discrepancy between the steady state output and the
input. The difference between the steady state output and the ideally desired output is called
steady state error. This specification is sufficient to give us an idea of the steady state response of
the system. Normally, in any system the steady state error is evaluated for certain common test
signals i.e. step, ramp and parabolic signals.
(b) Over shoot: The over shoot is an indication of the largest error between the actual output and the
desired output. Over shoot is normally expressed as percentage defined as:
%over shoot = max overshoot/final desired value x 100 C(t
Unit
) Max
step
1.05=10 Overhead
1.0=100
5%
%0.95=95
%
0.90=90
% 0.5
00.1
0
(c) Rise time: It is defined with respect to the step response ofOa system and donatedt by t r and is
Td T
defined as the time taken by the output to reach 10% to 90% of the step tr response of a system. Final
p
desired value. Sometimes t r is defined also as the reciprocal of the response atsystem
Fig: Unit step response of a the instant the
response is equal to 50% of its final value.
(d) Time delay: with reference to step response of a system, time delay is defined as the time required
for the response to reach 50% of its final value. The delay time is shown in the Fig.
6

(e) Settling Time: Theoretically the output of the system will become equal to its final desired value
only after a very large time. But practically the transient response lasts only for a very small time.
Settling time (Ts) is defined as the time required for the response to reach and thereafter to stay
within a specified percentage of its final value. The specified percentage is usually 5% or 2%
4. Standard Test Signal r (t)
(i) Step signal r(t) = Au(t); where u(t) = 1; t>0
& R(s) = A/s = 0; t<0 A
0 t
r(t)
(ii) Ramp signal: r(t) = At; t>0 & R(s) = A/s2
=0, t<0
0 t
r(t)

(iii) Parabolic signal: r(t) = At2/2; t>0 & R(s) =A/s3


= 0; t<0 t
0

r(t)
(iv) Impulse signal: δ (t )=0; t ≠0 ∫ ∫ ( t ) dt =1, where ϵ →0
ϵ
A
& δ (t) = ů (t); L [ δ ( t ) ]=1 R (s)
Impulse response of system with transfer function c(s)/R(s) = G(s) is given 0 1/A t
By (s) = G(s) R(s) = G(s)-1=G(s) it self
∴ c ( t )=L−1G(s) = g (t) i.e. actually impulse
Response of a system = g (t) = L−1 [G ( s )] = Weighting function.

5. Time Response of 1st Order System:

R(s) 1 C(s) C ( s) 1
=
Ts+1 R (s ) Ts+1

1 1
(i) Response to the unit step input is is R ( s )= ∴ ( s ) = R( s)
s Ts+1
1 1 T −t /T
I. e C(s) = = − ; ∴ c (t )=1−e
S ( Ts+1 ) s Ts+1

e. (t)
1.0
c ( t ) =1−e−t /T
&c(t)
The initial slope of thecurrect t=0 isGiven
−t
e (t )=e /T dc
By /t=0 = 1/T e t /T /t=¿0
o t dt
T st
Fig. unit – step response of a 1 order system.

(Where, T is known as the Time Constant of the system).


(Small time constant means faster response)
e (t) = r(t) – c(t) = e−t /T
7

= error response
Es s = steady state error = Lim e(t) = 0
t∞

c (t)
0.018=1.8% tolerance limit
st
1 Order system 1

1/a=Time constant 4/a t


ts

Step response of a first order system T=1/a = Time constant


Settling Time = Ts = 4/a = 4T
−t
e T =0.02 ≈ 0.018
1
¿ +t =0.02≈ 0.018
eT
¿
e
1
+t
T
=
1
=
1 −t
∨ =l oge
0.02 0.018 T
1
0.02
=l n
1
0.02 ( ) ( )
∨ln ⁡(
1
0.018
)

t
¿ =3.91∨t s =3.91T ≈ 4 T But for 5 % tolerance limit , t s=3 T
T

(ii) Response to the unit – ramp input: i.e. R(s) = 1/s2


2
1 1 T T
C(s) = 2 = − +
s (Ts+ 1) s
2
s Ts+1

∴ c ( t )=t −T ( 1−e )∧e ( t )=r ( t )−c ( t ) =T (1−e


−t
T −t /T
)
lim e ( t )
∴ e ss = =T
t→∞
c (t)
Reducing the system - time constant, improves the r (t) r(t)=t Steady state error=T
Speed of response and also reduces the steady state error.

c (t)

(t)

Fig: Unit-ramp response of a 1st order system


6. Time response of Second order system:
The first order system is simple and hence easy to work with unfortunately, in practice one is forces
to deal with higher order systems. Basic to understanding the response of higher-order systems and to the
problem of designing compensation for these systems is an understanding of the response of a second
order system. A second order system will be characterized by a characteristic equation of second degree.

s2+2α s+ (α2+ω2) = 0 ………….... (1)


The above equation can also be written as
8

s2 +2 δ ω n s+ ω2n=0 …………..… (2)


Comparing equations (1) & (2), we have
α =δ ωn …………….. (3)
2 2 2
α + ω =ωn …………….. (4)
Before proceedings with the analysis, at this point it is advisable to define some of the important
quantities.
(i) Damping ratio: The quantity δ introduced in equations (2) and (3) is called the damping ratio of
the system.
(ii) Damping constant: The actual damping of the system is described by damping constant, δ ω n.
(iii) Un-damped natural frequency. Un-damped means no damping and therefore equation (1) with
α=0 is the characteristic equation for an un-damped system. s2 +ω 2=0 ………….(5)
It is clear from Eq. (5) that the un-damped natural frequency of the system ω n is equal to the
frequency which is the solution of above equation.
(iv) Conditional frequency: It is the actual frequency of the system denoted by ω. From (4);
2 2 2
α + ω =ωn
2 2 2 2 2
Or ω =ω n−α =ω n (1−δ )
Hence, ω=ω n √ 1−δ 2 ⋯ ⋯ ⋯ (6)
Now, the solutions of the characteristic equation (2) are:
s1 , s2=−δωn ± ω n √ δ −1 If δ is > 1 ⋯ ⋯ ⋯ (7)
2

And s1 , s2=−δωn ± j ω n √ 1−δ 2 if δ is < 1 …………… (8)

SPECIFICATION OF A SYSTEM
If δ>1, roots are expressed by Eq. (7), However, for δ<1, roots are given by:
s1 s 2=−δωn ± jωn √1−δ 2 …………… (8)
As is clear, the nature of the roots of the characteristic equation and hence the system behavior, depends
on the damping radio δ. It is observed that for
δ >1 , s1 s 2=−δωn ± ωn √ δ −1 …………. (9)
2

δ=1 , s1 s 2=−ω n …………. (10)


δ <1 , s1 s 2=−δωn ± jωn √1−δ 2 ………….. (11)
δ <0, s1 s 2=± jωn ………….. (12)
Because of the dependence of the roots the Characteristic Equation on the damping ratio, the different
cases are summarized in the table below:
TABLE
Damping RatioNature of Roots System Behavior
δ>1 Real and Different over damped
δ=1 Real and equal critically - damped
δ<1 Complex conjugate under damped
δ=0 Purely Imaginary un-damped

0<δ<1 Jω
-j ω
δ<1 δ=0
X
δ>1 δ>1 δ∞
9 δ<1
δ>1 X δ=0

δ=1
Fig.: Variation of roots with δ
θ
δ

If ω n is constant, the roots of characteristic equation vary as a function of δ as shown in the above figure.
It is clear that as δ varies from 0 to ∞, the location of the roots of the character equation will move away
from the imaginary axis along a circular path of radius ω n the roots will meet at the points s = - ω n when
δ=1 and then separate and travel along the real axis forwards zero and infinity. For complex roots, ω n is
the radial distance from the roots to the origin, i.e.
ω n=√ α 2+ ω2 ……………………. (13)
Also, the damping ratio is equal to the cosine of the angle between the radial line and the negative real
axis, i.e.
δ = Cos θ or θ = cos-1δ (14) (i.e. from eqn.8)
The transfer function of a typical second order system is given as:
C( s) ω 2n
G ( s )= = 2 ……………………… (15)
R( s) S + 2 δ ωns +ω 2n
In a general case, suitable multiplicative factor can be used.
For the step response, r (t) = u-1(t) or R(s) = 1/s. Hence,
ω 2n K
C ( s) = 2 s 2 ……………………… (16)
s( s + 2δωn +ω n)

K Illustration

c(s) ωn2
The transfer function of a typical second order system is given by G ( s )= = 2 ……………….(16)
R(s) ( s + 2 δ ωn s +ω 2n)
And R( s) = L [u (t)] = 1 / s
ω2n A Bs+c
C ( s) = = + 2
2
s( s + 2δ ωn s +ω ) 2
n
s ( s +2 δ ωn s+ ω2n )
Or ω n= A ( s +2 δ ωn s+ ωn ) +( Bs+c ) s
2 2 2

2 2 2
¿ As + A 2 δ ωn s + A ω n+ Bs +cs
2 2
Equating the term by term both sides of above Equation, Hence ω n= Aωn → A=1
s ( A 2 δωn+ c ) =0 → c=−2 δωn
2
s ( A +B )=0 → B=− A=−1
1 −s−2 δωn
Hence C ( s ) = + 2
s s +2 δωns + ω2n
1 s+δωn δωn
¿ − 2 − 2
s s +2 δωn s+ ωn s + 2 δωn s +ω 2n
2

1 ( s +δωn ) δωn
¿ − −
s ( s+δωn ) +ω n ( 1−δ ) ( s+ δωn)2 +ω 2n ( 1−δ 2 )
2 2 2

[ ∵−s−2 δω=−s−δ ωn−δ ωn ∧¿∵ s2 +2 δ ωn s+ ω2 n=(s +δωn)2+ ω2 n(1−δ2 )]


10

(δ ω n) ωn √ 1−δ
2
1 ( s+ δωn )
¿ − −
s ( s+δωn )2 +( ω √ 1−δ 2)2 ω 1−δ 2 s +δ ω 2 + ω 1−δ 2 2
n n√ ( n) ( n√ ) [ ]
sin ( ω n √ 1−δ 2 ) t . δ . e−δ ω t
n

Hence C (t) = L
−1
[ c ( s ) ] =1−cos ( ωn √ 1−δ ) t . e
2 −δ ωn t

( √1−δ 2)
e
=1− ¿
√ 1−δ 2
Let – δ =cosθ∧√ 1−δ 2=sinθ
e−δ ω t
[sinθ cos θ ( ωn √ 1−δ ) t−cos θsin ( ωn √ 1−δ ) t] …….. (17)
n
2 2
Then C ( t )=1−
( √1−δ ) 2

−δ ω n t
¿ 1+
e
[ sin ( ω √1−δ ) t . cosθ−cos ( ω √1−δ ) t . sinθ ]
2 2

√1−δ 2 n n

sin ( ωn √ 1−δ 2 ) t−θ where ; θ=tan−1 √


−δ ω n t 2
¿ 1+
e
[ ] 1−δ
√1−δ
−δ ω t
2 −δ
e n

sin ( ωt−θ ) ; where ω=ωn √1−δ


2
¿ 1+
√1−δ 2

[let−δ =cosθ∧ √1−δ =sinθ]


2

The quantity inside the parenthesis in equation (17) can easily be written in terms of a shifted
sinusoid using standard trigonometric identities. Output can then be expressed as:
e−δω
n t
C ( t )=1+ sin(ωt −θ) ………. (18)
√1−δ 2
Where, ω=ω n √ 1−δ 2 ⋯ ⋯ ⋯ (19)δ

And θ=tan
−1 √1−δ 2 ………….. (20)
−δ
The output of the system expressed by the above equations can be plotted for various values of δ. In the
figure below, this curve is plotted for several values of δ as a function of normalized time, ω n t.

1.6

1.4

1.2

1.0
C step

0.8
0.6
0.4
0.2
0
2 4 6 8 10 12 14
ω nt
Fig: Step response of a second order system.
−δω
It may be readily observed that the final value of c (t) is 1 because of the factor e n t in the second term of the
expression for c (t). Hence, there is no steady state error and it is stated that a second order system also tracks a unit
step input with no error:
11

It is also clear from the above Fig. that no overshoot occurs for δ≥1. However, when δ<1 and overshoot occurs. To
d
find out the value of this overshoot, we first find out the maximum output which occurs when C ( t )=0.
dt

Or, from equation (18)


−δω
d
dt
e−δω t
[
1+ n 2 sin ⁡(ωt−θ) =0
√1−δ ]
en t
[ ω cos ( ωt −θ ) + {sin ( ωt−θ ) }(−δωn )]=0
√1−δ 2
Or ω cos ( ωt−θ ) + { sin ( ωt−θ ) } (−δωn ) =0
Or ω n √ 1−δ 2 cos ( ωt−θ )−δ ωn sin ( ωt−θ ) =0=ωn sin ( ωt−θ ) =0=ωn sin ωt
i. e. ω n sinθ cos ( ωt−θ ) + ωn cos θ sin ( ωt−θ )=ω n sin ( θ+ωt −θ ) ¿ ωn sin ωt=0
[∵−δ =cos θ∧√ 1−δ =sinθ ] and sin (A+B) = sin A. Cos B + Cos A. sin B
2

Using ω=ω n √ 1−δ 2


ω n sin ωt =0
ωt=ωn √ 1−δ 2 t=nπ
Or

Hence t= ……………………. (21)
ωn √ 1−δ 2
The maximum or minimum value of output therefore occurs at a time t p, given as:

t p= …………………………. (22)
ωn √ 1−δ 2
At this value of time,

c (t) Max or Min. ¿ 1+


√1−δ
1
2
exp
(√ )
−nπδ
1−δ
2
. sin( nπ−θ)

¿ 1+
(−1 )n−1
√ 1−δ 2
exp
(√ )
−nπδ
1−δ
2
sin ¿ ¿ ………………… (23)

Making use of a trigonometric identity as illustrated in Fig. 9.5;

(
sin tan −1
δ )
√1−δ2 =√ 1−δ 2 ……………………… (24)

1 √ 1−δ 2
θ
δ
Fig 9.5: Illustrating a trigonometric identity

{√ }
π
n−1 −n δ
c (t) max or min ¿ 1+(−1) exp 2
……………………… (25)
1−δ
An examination of Eq. (25) reveals that:
(1) For odd values of n, c (t)>1 and is therefore a maximum overshoot obviously can occur at these points.
(2) For even values of n, c (t) <1 and is therefore a minimum. At these points, an under shoot occurs.
(3) For odd values of n, maximum value of c (t) decrease as n increase. Therefore the maximum of the maxim as
occurs only at n=1. At this value of n, the time can be found from Eq. (22) as
π
t p= ……………………. (26)
ωn √ 1−¿ δ 2 ¿
The maximum value of c (t) is equal to:

Max c ( t ) =1+ exp {√ }−πδ


1−δ
2 …………………. (27)

Maximum overshoot
12

¿ exp
(√ )
−πδ
1−δ
2

Hence percentage overshoots

¿ 100 exp
(√ )
−πδ
1−δ
2 …………………………. (28)

Normalized value of t p and the percentage overshoot are plotted in Fig. It may be noted that the percentage
overshoot a function of δ only, i.e. it does not depend upon ω n. It is also seen from Fig. that a damping ration of 0.7
corresponds to an overshoot of approximately 7%, whereas a damping ratio of 0.3 corresponds to an overshoot of
approximately 35%, for most systems in practice, an overshoot in this range is found tolerable or even desirable. This
for practical purpose the range of δ of interest is. 0.3 ≤ δ ≤ 0.7
5.4 1.2

Per unit over shoot


5.0 1.0

4.6 0.8
ω nt p

4.2 0.6

3.8 0.4

Mp
3.4 0.2

3.0 0
0 02 0 02 04 06 08 10
04 06 08 10
Damping
Fig: (a)factor δ Damping
Fig. (b) factor δ ->
Fig: Time to peak and percentage overshoot as a function of δ
The range of desirable pole location in this plane for second order system is thus restricted
j to the shaded arrears of Fig. It is observed from equation (34) that for a fixed value of δ, t p
ω varies inversely as ω n. Thus a high or large value for ω n means a small t p and we have a
quick responding system. On the other hand, a small value of ω n means a large t p and
17⁰ consequently a slow responding system. From Fig. 9.3 we see that ω n gives the distance of
45⁰ the poles from the origin. Thus, in general the farther away from the origin the system pole
o
is located the faster the system responds.

To have an idea about the settling time for the second order system, we see from (26) that
the magnitude of the oscillations decreases as e –δ ω n. In the same manner as the time
constant (defined for first order systems, we now define the time constant for the second
Fig: Region of desirable order system by T=1/δ ω n ………………. (29)
pole locations
In time duration of four time constants, the output will settle to within 1.8 percent of its final
value. Thus, the settling time for the second order system may be defined by.

Ts= 4τ=4/δ ω n ………….. (30)

Example-1: The system illustrated in Fig.1 consists of a unity feedback loop containing a minor rate feedback loop.

(a) Without any rate feedback (b=0) determine the damping factor (ratio) natural resonant frequency and
overshoot of the system.
(b) Determine the rate feedback constant which will increase the equivalent damping factor of the system to
0.8. Determine the over shoot of the system in this case also.

Fig.1: Block diagram for the


13

Solution (a) For b=0, G(s) = 16/s(s+4) and H(s) = 1


Therefore the system characteristic equation is:
16
1+ =0 Or s2 +4 s +16=0↔ s2 +2 δ ω n s+ω 2n=0 …………………… (31)
s ( s+ 4 )
Comparing equation (31) with standard quadratic characteristic equation:
2
2 δωn=4∧ωn =16
Solutions of the above relations, ω n=4∧δ =0.5
Hence damping factor = 0.5 = damping ratio
Natural frequency = 4 rad. / sec

Percentage overshoot ¿ 100 exp {√ −0.5 π


1−0.25 }
¿ 16.4 percent
(b) With rate feedback in the circuit,
16
s ( s+ 4 )
G ( s )=
16 bs
1+
s ( s +4 )
16
¿
s ( s+ 4 ) +16 bs
Hence the characteristic equation for the system is:
16
1+ =0
s ( s+ 4 ) +16 bs
2 2 2
Or s + ( 4 +16 b ) s+16=0 ↔ s +2 δ ω n s+ ωn =0 ……………….. (32)
2
Now ω n=16∨ωn =4
2 δωn=4+16 b
Hence δ =0.5+ 2b=0.8
Therefore, b=0.15 ………………….. (33)
The percentage overshoot

¿ 100 exp
{√ }
1−δ 2
−πδ

¿ 100 exp
{
−0.8 π
√ 1−0.64
=6.8 %
}
Example-2: Figure (a) shows a mechanical vibratory system. When 8.9 N of force is applied to the system, the mass
oscillates, as shown in Fig, (b). Determine the value of M, F and K for the system from this response curve.

X (t) 0.0029
K (Meter)
Y 0.03
M

F
1 2 3 4
Fig: (b) Step response

Fig: (a) Mechanical system


14

Solution: The transfer function of the mechanical system is readily determined as:

X (s) 1 1
= =
Y ( s) Ms 2 + Fs+ K F K ……….. (34)
Ms 2+ s+
M M
Now, the force applied being 8.9 N
Y(s) = 8.9/s
8.9
Hence X (s)= …………….. (35)
s¿¿
The steady state value of the output x (t) is
8.9
x ss =lim ¿ t → ∞ x ( t )=lim ¿ s →0 sX ( s )= =0.03
K
8.9
Hence K= =297 N /m …………….. (36)
0.03
0.0029 −πδ
From the system response percentage overshoot: ¿ 0.03 ×100=9.67 per cent=100 e
√1−δ 2
This value of percentage overshoot corresponds to a value of damping ratio, δ=0.6
The time for the occurrence of the overshoot is:
π π
t p= =
ωn √ 1−δ ω n √ 1−(0.6)2
2

π 1
¿ .
0.8 ωn
π
As this value from the figure is known to be 2s= =2
0.8 ωn
π
Or ω n= =1.96 rad /sec
2× 0.8
From the system characteristic equation,
K
ωn =
2
M
K 297
Or M= = =77.3 kg
ω n (1.96)2
2

F
Also 2 δ ω n=
M
F=2 δ ω n = (0.6) (1.96) (77.3)
= 181.8 ns / m.
Hence, the system constants are:
F=181.8 Ns/m
M=77.3Kg
K=297 N/ms
Example-3: In a D.C. position control servomechanism, the load is driven a by a motor supplied with
constant armature current. The motor field current is supplied from a D.C. amplifier, the input to which is
the difference between the voltages obtained from input and output potentiometers.

S
If Tachometer
Motor
K P (θ ¿ ¿ R−θC )=V E ¿
TM
+KA
θM
15
Fig. D. C Position control servo

I a Constant
θR θC Load, j, f

The load and the motor together have a M.O. I. of J=0.4 kgm 3 and the viscous friction is f=2 Nm/rad./sec.
Each potentiometer constant is KP=0.6 V/rad. The motor develops a torque of K T = 2 N-m per ampere of
field current. The field time constant is negligible.

(a) Derive the system characteristic equation (when S is open) and find the value of the amplifier gain
KA (in amperes output per volt input) to give a natural frequency of 10rad/sec. (without
Tacho-generator feedback)
(b) A techno generator of negligible inertia and friction is connected in the system to improve the
damping. Determine the taco generator constant (in V/ rad. /sec) to give critical damping for K A = 5

Solution: Given the signal flow graph of the system.

1 1

Fig: SFG for the system of the example

With switch s open i.e. taco generator not in the loop,

θM 1 1
=G ( s )=K P K A K T
θR Js +f s
KP K A KT
Or G ( s )= ………………. (37)
s( js+f )
The system characteristic equation is thus
1+G(s) = 0
Or s ( Js +f )+ K P K A K T =0
2
Or Js + fs+ K P K A K T =0 ……………… (38)
2
K K K
Now, ω n = P A T =(10) ; ω n=10
2
J
J . ωn
2
0.4 ×100
Hence K A = =
K P . KT 06 ×2
Or KA = 33.3 amp/volt
(b) With tachometer, i.e. switch S closed,
1
K A . KT
Js+ f 1
G ( s )=K P .
1 s
1+ K A . K T . Kt
Js+ f
16

K P. K A . KT
G ( s )= ……………….. (39)
s (Js + f + K t . K A . K T )
Where Kt is the taco meter constant:
Now the characteristic equation is
1+G(s)=0
Or s(Js+ f + K A . K T . K t )+ K P . K A . K T =0
2
Or Js +( f + K A . K T . K t ) s + K P . K A . K T =0……………….. (40)

It is observed that
KP. KA.KT
ωn = 2
J
f +K A . KT . Kt
And 2 δωn=
J


f + K A . KT . Kt 1 J
Therefore, δ = . .
J 2 KP . K A . K T
Now for critical damping, δ=1
2 √ K P . K A . K T j −f
Kt= =0.11V /rad / sec
K A KT

7. Steady state errors and error constants: Steady state errors constitute an extremely important aspect of
system performance, for it would be meaningless to design for dynamic accuracy, if the steady output
differed substantially from the desired value for one reason or another. The steady state error is a measure
of system accuracy. These errors arise from the nature of inputs, type of system and from nonlinearities of
system components such as static friction, backlash, etc. these are generally aggravated by amplifier drifts,
aging or deterioration. As discussed in the introduction to this chapter, the steady state performance of a
stable control system is generally judged by its steady state error to step, ramp and parabolic inputs.

Consider a unity feedback system shown in Figure. The input is R(s) the output C(s), the feedback signal
B(s) and the difference between input and output is the error signal E(s). From fig. we see that

C (s) G (s ) G ( s)
= ∨C ( s ) = R (s ) R(s) + E(s) C(s)
R ( s ) 1+G ( s ) 1+G ( s ) G(s)
G (s ) -
C ( s ) =E ( s ) G ( s )∨ R ( s )=E ( s ) G( s) B(s)
1+G ( s )
1 Fig: Unity feedback system
E ( s )= R( s) . . . . (5.21)
1+G ( s )
The steady state error e s s may now be found by use of the final value theorem as follows.
sR ( s )
e ss =lim ¿ t → ∞ e ( t ) =lim ¿ s → 0 sE ( s )=lim ¿ s →0 …………………. (41)
1+G(s )
Equation (41) shows that the steady state error depends upon the input R(s) and the forward transfer function G(s).
The expression for steady – state errors for various types of standard test signals are derived below.
17

A. STATIC ERROR CONSTANTS

1. Unit Step Input


Input r (t) = u (t)
R(s) = 1/s
(From eqn. 5.21)
1 1 1
e ss =lim ¿ s → 0 = = , where K p = G (0) is defined as the position error constant.
1+G( s) 1+G(0) 1+ K p
2. Unit Ramp (Velocity) Input
Input r(t) = t or R(s) =1/s2
(From eqn. 5.21)
1 1 1
e ss =lim ¿ s → 0 =lim ¿ s → 0 = , where K = lim ¿ s → 0 sG(s) is defined as the
sG( s) K v
v
s+ sG(s)
velocity error constant

3. Unit Parabolic (Acceleration) Input


Input r (t) = t2/2 or R(s) = 1/s3
(From eqn. 5.21)
1 1 1
e ss =lim ¿ s → 0 =lim ¿ s → 0 2 = ,
2
s + s G( s)2
s G(s ) K a
Where K a =lim ¿ s → 0 s2 G(s) is defined as the velocity error constant
TYPES OF FEEDBACK CONTROL SYSTEM:
The open loop transfer function of a unity feedback system can be written in two standard forms – the time
constant form and the pole – zero form. In these two forms, G(s) is as given below.
K ( T z 1 s+1 ) ( T z 2 s +1 ) …
G ( s )= n (Time constant form) . . . . . . (5.25)
s ( T p 1 s+ 1 )( T p 2 s +1 ) …
'
K ( s+ z 1 ) ( s + z 2 ) ….
¿ n (Pole zero form) . . . . . . . . . . . . . . .(5.26)
S ( s+ p1 ) ( s+ p2 ) ...
Π zi
' i
The gain in the two forms is related by K= K Π . . . . . (5.27)
j pj

In the gain relation Eqn. (5.27), the two forms of G(s), it is sufficient to obtain steady state errors in terms of
the gains of any one of the forms. We shall use the time constants form in the discussions below. Equation
5.25 involves the term s n in the denominator which corresponds to number of integrations in the system. As
s tends to zero, this term dominates in determining the steady state – state error. Control systems are
therefore classified in accordance with the number of integrations in the open loop transfer function G(s) as
described below.

K
K () 1 K ()
Type – O – System: G ( s )= 0
, E ( s )= . R ( s ) ;G ( s )= 0
s () 1+G ( s ) s
(i) Unit step input: i.e. r(t); R(s) = 1/s,
e ss sR ( s ) 1 1 1 1
=lim ¿ t → ∞ e ( t )=lim ¿ s → 0 sE ( s ) =lim ¿ s → 0 =lim ¿ s →0 = =
( position) 1+G ( s ) 1+G(s) K ( ) 1+ K 1+
1+
()
lim ⁡G(s )
Where K p = G (o) = K: Position Error Constants = =K
s →0
18

1
(ii) Unit Ramp Input: i.e. r (t )=t , R( s)= 2
s
e ss lim ⁡s sR ( s ) 1 1
=lim ¿ t → ∞e (t )= E ( s )=lim ¿ s → o =lim ¿ s →o =lim ¿ s →o =
(velocity) s →0 1+G ( s ) s(1+G ( s ) ) (s+ G ( s ) )

lim ⁡sG ( s )
; K v =Velocity Error Constanant=
s→o

(iii) Unit Acceleration Input: i.e. r(t) = t2/2; R(s) = 1/s3,


e ss sR ( s ) 1
=lim ¿ t → ∞ e ( t )=lim ¿ s → 0 sE ( s )=lim ¿ s → o =lim ¿ s → 0 2 =lim ¿ s → 0=
( Accleration ) 1+G ( s ) s ( 1+G ( s ) )

K () 1 K ()
2. Type – 1 – System: G ( s )= 1
; E ( s )= R ( s ) ;G ( s ) = 1
S () 1+G ( s ) S ()
(i) Unit Step Input: i.e. r (t)=u(t); R(s)=1/s,

e ss sR ( s ) 1 1 1
=lim ¿ t → ∞ e ( t )= lim ⁡ sE ( s )=lim ¿ s → o =lim ¿ s → 0 =lim ¿ s →0 = =0
( position ) s→0 1+G ( s ) 1+G ( s ) K () 1+ ∞
1+
s ()
(ii) Unit – Ramp Input: i.e. r(t) =t; R(s)=1/s 2
e ss sR ( s ) 1
=lim ¿ t → ∞ e ( t ) =lim ¿ s → 0 sE ( s ) =lim ¿ s → o =lim ¿ s → 0
( velocity ) 1+G ( s ) s¿¿
(iii) Unit Acceleration Input: i.e. r (t)=t2/2; R(s) = 1/s3
e ss sR ( s ) 1 1
=lim ¿ t → 0 e ( t )=lim ¿ s → 0 sE ( s )=lim ¿ s → o =lim ¿ s → 0 2 =lim ¿ s → 0=
( Accleration ) 1+ G ( s ) s ( 1+G(s) )
s2 1+
[K
S
K () 1 K ()
3. Type – 2 – System: G ( s )= 2
; E ( s )= R ( s ) ; G ( s )= 2
s () 1+ G ( s ) s ()
(i) Unit Step Input; i.e. r(t)=u(t); R(s)=1/s
e ss sR ( s ) 1 1 1
=lim ¿ t → ∞ e ( t )=lim ¿ s → 0 sE ( s )=lim ¿ s → o =lim ¿ s → 0 =lim ¿ s → 0 = =0
( Position ) 1+G ( s ) 1+G( s) K () ∞
1+ 2
s ()
(ii) Unit Ramp Input: i.e. r (t) = t, R(s) =1/S 2,

e ss sR ( s ) 1 1
=lim ¿ t → ∞ e ( t )=lim ¿ s → 0 s E ( s )=lim ¿ s → o =lim ¿ s → 0 =lim ¿ s → 0 =

( )
( Velocity ) 1+G ( s ) s (1+ G ( s ) ) K ()
s 1+ 2
s ()
(iii) Unit Acceleration Input: i.e r(t)=t 2/2 ; R(s) = 1/S3;
e ss sR ( s ) 1 1
=lim ¿ t → ∞ e ( t )=lim ¿ s → 0 sE ( s )=lim ¿ s → o =lim ¿ s → 0 2 =lim ¿ s →0
( Accleration ) 1+G ( s ) s [ 1+G(s ) ]
[
s 2 1+
K(
s2(
Thus K p = Position Error Constant = lim ¿ s → o G(s) = G (o)

K v = Velocity Error Constant = lim ¿ s → o sG(s)


19

Ka = Acceleration Error Constant = lim ¿ s → o s2G(s)


B. Generalized Error Series (i.e. Dynamic Error Series) and the Dynamic Error Coefficient.

Fig. A unity feedback system

R (s ) 1
E (s) ¿ = [ R ( s ) ] ------ (1)
1+G ( s ) 1+G ( s )

t
e (t )= ∫ we ( τ ) r ( t−τ ) dτ --------- (2) (Using Convolution Integral)
−∞
1
Here, w e ( τ ) is the inverse Laplace Transform of W e ( s )=
[ 1+G(s) ]
If the first derivatives of r (t) exist for all values of τ , the function r ( t−τ ) can be expanded into Taylor series; that is
' τ 2 '' τ 3 ' ''
r ( t−τ ) =r ( t )−τ r ( t ) + r ( t ) − r ( t ) +¿……………. (3)
2! 3!
Where the primes (‘) denote time derivatives. Since r(t) is zero for negative time, the limit of the convolution integral
may be taken from o to t; substituting (3) in (2) we get,
t
e (t )=∫ ωe ( τ ) ¿
o
t t t 2
τ
¿ r (t )∫ ω e ( τ ) dτ −r ( t )∫ τ ωe ( τ ) dτ+ r ' (t)∫
' '
ω ( τ ) dτ .... (4)
o 0 o 2! e
The steady state error is obtained as:
t ∞
e ss =lim ¿ t → ∞ e ( t ) =lim ¿ t → ∞ ∫ ωe ( τ ) r (t −τ ) dτ=¿ ∫ ω e ( τ ) r s ( t−τ ) dτ ¿
o o
∞ ∞ ∞ 2
τ
¿ r s ( t )∫ ω e ( τ ) dτ −r s ( t )∫ τ ω e ( τ ) dτ +r ss (t )∫
' ''
ω ( τ ) dτ ………… (5)
o o o 2! e
Where r s (t) denotes the limit of r (t) as t approaches infinity; i.e. the steady state part of r (t)
If we define:

C o=∫ ω e ( τ ) dτ
o

C 1−∫ τ ω e ( τ ) dτ ---- (6)
o

C 2=∫ τ ωe ( τ ) dτ
2


C n=(−1 )∫ τ ω e ( τ ) dτ
n n

o
Then, Eqn. (5) can be written as:
C2 ' ' ' Cn n
e ss =C o r s ( t ) +C 1 r s ( t )+
r s ( t ) + r s ( t ) +… … (7)
2! n!
C ,C
Here the coefficients, o 1 2, C … … C n are defined as the generalized error coefficients (i.e. also dynamic error
coefficients).
20


W e ( s )=∫ ω e ( τ ) e
−τs
dτ ---------- (8) Which is Laplace Transform of ω e ( τ )
o

Taking the limit of Eqn. (8) as approaches to zero, we have



lim ¿ s → 0 C o e−τs=C0 =lim ¿ s → 0W e ( s )=lim ¿ s →0 ∫ w e (τ ) e
−τs
dτ ……………. (9)
0

W e ( s )=L[w e ( τ ) ]

Taking the derivative of W e ( s ) w.r.t.s yields,



d W e (s)
=−∫ τ ωe ( τ ) e dτ=C 1 e ---------- (10)
− τs − τs
ds o
d W e ( s)
Hence C 1=lim ¿ s → 0 --------- (11)
ds
n
d W e (s)
Similarly c n=lim ¿ s →0 n
---------------- (12)
ds
i. e. C o=lim ¿ s → o W e ( s )

d W e ( s)
C 1=lim ¿ s → o
ds

d 2 W e ( s)
C 2=lim ¿ s → o
ds2
n
d W e (s)
C n=lim ¿ s → o n
ds
Note: For non-unity feedback system G(s) is replaced by G(s) H (s)
The advantages of using the generalized error coefficients are summarized as follows:
(1) The generalized error coefficients provide a simple way of determining the nature of the response of a feedback
control system to almost any arbitrary input.
(2) The generalized error coefficients lead to the calculation of the complete steady-state response without actually
solving the system differential equation.
K
4. Example: In a unity feedback control system the open look transfer function is given as G ( s )= find the
s +1
a2 t 2
steady state error using generalized error coefficients for the input r(t) = (a 0 + a1 t+ ¿ u(t ).
2
1 s+1
Solution: For this system, W e ( s )= =
1+ G(s) s+ K +1
lim W e ( s ) ⁡ 1
C 0= =
s→0 1+ K
lim ⁡ d W e ( s) lim ⁡ 1 [ s+ K + 1 ] −[ s +1] K
C 1= = =
s →0 ds s→0 (s+ K +1)
2
(1+ K )
2

2
d W e (s) −2 K
C 2= lim ⁡ =
s →0 d s 2 (1+ K )
3

' ''
e ss (t )=C 0 r s ( t ) +C 1 r s (t ) +C 2 r s ( t ) +⋯
a2 t 2 (If transient part of r(t) is present
Given r (t) = (a0 + a1 t+ ¿ u(t ); r’ (t) = (a1+ a2t) u(t) = r s’(t) ; r’’(t) = a2 u(t) = r s s’’(t)
2 that will be zero in taking the limit
t ∞ , or else r s ( t )=r (t)
21

Hence,
' ''
e ss (t )=C 0 r s ( t ) +C 1 r s (t ) +C 2 r s ( t ) +⋯
1 a2t 2 K −2 K
= (a0 +a1 t+ )u (t)+¿ 2 (a1+ a2t) u (t) + 3 a2 u (t)
1+ K 2 (1+ K ) (1+ K )

Or e ss (t )=
[ 1+ K
1
a0 +
K
(1+ K )
a−
2 1
K
(1+ K) ][a +
3 2
a1
+
a1 K
1+ K ( 1+ K ) ]2
t+
a2
(1+ K )3
5. Example
The system illustrated in Fig.2: below is a unit feedback control system with a minor feedback loop (Output
derivative feedback).
(a) In the absence of derivative feedback (a=0), determine the damping factor and natural frequency. Also
determine the steady – state error resulting from a unit-ramp input.
(b) Determine the derivative feedback constant of which will increase the damping factor / damping ratio of the
system to 0.7. What is the steady – state error to unit-ramp input with this setting of the derivative feedback
constant?
(c) Illustrate how the steady-state error of the system with derivative feedback to unit ramp input can be
reduced to the same value as in part (a), while the damping factor is maintained at 0.7 (by increasing gain)

R ( s +¿
) +¿ 8 C ( s)
s (s +2)
−¿ −¿
as

Fig: 2
Solution:

8 8
(a) With a=0, the Ch. Eqn. is s(s+2)+8=0: G (s) = ; Ch. Eqn.=1+G ( s )=1+
s (s +2) s (s +2)
2 2 2
s +2 s +8=0↔ s +2 δ ω n s+ ωn =0
Hence ω n=√ 8=2 √ 2 rad /sec
2
2 δωn=2 or δ= =0.353
2× 2 √ 2
lim ¿ lim ¿ 8
System K v= s → 0 s G ( s )= s →0 =4 ¿ ¿
( s+2 )
1 1
Hence e s s (to unit ramp input) = K = 4 =0.25
v

( )
8 /s (s +2)
(b) With derivation feedback, the Ch. Eqn. is 1+G(s)=0 And Now G(s) = 8 as
1+
s( s+ 2)

( )
8/ s ( s+2) 8 s ( s +2 ) +8 as +8
¿ 1+ =0=1+ =
8 as s ( s +2 ) +8 as s ( s+ 2 )+ 8 as
1+
s (s +2)
2 2 2
Or s + ( 2+8 a ) s+8=0 ↔ s +2 δωns+ ωn=0
Hence 2 δ ω n=2+8 a∧ω n=2 √ 2
8 /s (s +2) 8
G ( s )= = ;∨a=0.245
Or 2 ×0.7 × 2 √ 2=2+8 a Now as 8 s( s+ 2+ 8 a)
1+
s( s+ 2)

8 8
System K v= lim ¿ s → 0 s G ( s )= [s+2+ 8 a] = 2+ 8 ×0.245 =1/0.495
22

1 2+8 × 0.245
∴ e ss = = =0.495
Kv 8

(c) Let the gain of 8 in the forward loop be adjusted to a higher value K A. The new Ch Eqn. is
s2 + ( 2+a K A ) s+ K A =0 ↔ s2 +2 δωns+ω 2n
∴ ωn =√ K A ; 2 δ ωn=2+ a K A
Or 2 ×0.7 × √ K A =2+a K A ……………….. (1)
KA
System K v =lim ¿ s → 0 sG ( s )=
(2+ aK A )
1 2+ a K A
Hence e ss = = =0.25 -------- (2)
Kϑ KA
Solving Eqns. (1) and (2) we get, KA=31.36 and a = 0.186

Alternative Solutions:
Adding an amplifier of gain KA between the two summing points, the Characteristic
Equation becomes:
K A 8/ s ( s +2 ) KA
1+G ( s )=1+ =0 = 1+ 2
1+ 8 as / s(s+ 2) s + ( 2+ 8 a ) s
2 2 2
Or s + ( 2+8 a ) s+8 K A =0 ↔ s + 2δ ωn s +ω n
Hence ω n=√ 8 K A =2 √2 √ K A∧2 δ ωn=2+ 8 a
Hence 2 ×0.7 × 2 √ 2 √ K A =2+8 a ------------ (3)
System K v =8 K A /(2+8 a)=lim sG ( s )
s →0
Hence e ss =(2+8 a)/ 8 K A =0.25 ------ (4)
Solving Eqns. (3) and (4), we get
K A =3.92 ; a=0.73
Note: The alternative solution requires a small gain but a separate amplification stage.
23

VOL-2: K 2
Vol. 2, Module: 2 K2: Time Response of Positional Servomechanism:

i a Ra La K K b
i i f = constant
D.C.
e−¿ Amplifier
+ ¿¿
¿ e bM
J m Bm θm
Tm
N1
K s ( θ r−θ c ) =K s θc =e

V JL
θr θc
BL
θr −θc =θ e N2
K s θ e=e Error Detector = K s

Fig: Positional Servo Mechanism

Fig: State Transition Signal Flow Graphs of an Armature Control Servo mechanism:

System Description:

The system is to position a mechanical load with viscous friction and inertia. A Potentiometer Error –
detector with sensitivity Ks is used to measure the discrepancy between the reference and the controlled shaft. The
actuating signal is amplified by a D.C. amplifier, whose output is connected across the armature of D.C. servomotor.

Sensitivity of error detector = Ks = 1/57.3 volt/deg = 1vo/rad.


Grain of the D.C. amplifier = A (Variable)
Resistance in the armature of motor = Ra = 5Ω
Inductance in the armature of motor = La = negligible = 0
−3 ft 2
Inertia of motor = J m =10 lb− / sec
rad
Friction of Motor = Bm=negligible=0
Inertia of Load = J L=0.1lb−ft / rad /sec 2
Friction of Load = BL =0.1 lb−ft /rad /sec
24

N1 1
Gear ratio = ¿ n= =
N 2 10

ft
Motor Constant = ¿ K i=0.5lb−
amp
The Back EMF constant Kb is not given originally, but a definite relation exists between Kb and K i. In the English (i.e.
British) system, K i am given in lb-ft/amp and the unit of the back EMF constant kb is volt/rad./sec. With these units,
Kb and K i differ only by a constant ratio.
The mechanical power developed in the motor armature is:
1
p ( t ) =e b ( t ) .i a ( t ) watts= e ( t ) i ( t ) Hp … ..(1)
746 b a
But e b = K b w m (t)……. (2)
And T m (t) = K 1 I a (t)……… (3)
Or I a (t) = T m (t)/ K i……….. (4)
Putting (4) in (1) we get
p(t) = 1/746 e b T m/ K I ……. .(5)
Putting (2) in (5) we get
p (t) = 1/746 kb wm Tm/ K i H.P……..(6)
Also p (t)=1/550 Tm(t)wm(t) ………(7)
Equating (6) and (7) we get
Kb/746Ki T m (t)=1/550 Tm(t)wm(t)
Or 550 or or K i =0.7376 ………….. (8)
K i= k K b =1.36 k i
746 b
Given K i =0.5lb-ft/amp,

Hence Kb=1.36 K i = 1.36 x 0.5 = 0.68 volt/rad./sec

Derivation of Equations for drawing S.F.G: Initial conditions are assumed to be zero.
(1) Error Detector: θ e (t) = θ r (t)-θ c (t)θ E(s) = θ r (s)-θ c (s)……………(9)
e (t) = Ks θ e (t) E(s)=K s θ E (s)…………………………(10)
(2) D.C. Amplifier: ea(t)=A e(t)=AK s θ e(t)Ea(s)=AK s θ e (t)…………. 11)
(3) D. C. Servomotor (Armature Controlled):
Ra E a ( s )−Eb (b)
La
dia
dt (
=−R a i a ( t ) + ea ( t ) −e b ( t ) → I a ( s ) s +)La
=
La
…………. (12)

Eb ( s )=K b s θm (s) …………… (13)

Jme
d ωm ( t)
dt [
=−B me ω m ( t ) +T m ∵ T m ( t )=J me
d 2 θm (t)
dt 2
+ Bme

dt ]
……………………….. (14)

And T m ( t )=K i i a (t) ………… (15)


dθm ( t )
ωm (t) ………. (16)
dt
25

θc ( t ) =n θm (t )……. (17)
2 −3 1 −3 2
J me =J m +n J L=10 + ×0.1=2× 10 lb−ft /rad /sec
100
2 −3
Bme =B m +n Bl =10 lb−ft / rad /sec

1 1
s Ki s
Ks A n
θr (s ) 1 θe ( s) La
R
La ( )
1+ a 1 /s J me 1+
B me 1
.
J me s
s θc ( s)

−K b /La
−1

Ki 1
.

( ) ( )
Ks A Ra B n
θe ( s) s+ J s + me
La La me J me s θc ( s)
θr ( s ) 1

−K b /La
−1
Or

( )
Ks A K in
×
1

( ) ( )
La Ra B s
θr (s ) θe ( s) s+ J me s + me
Hence La J me θc ( s )
Kb Ki
1+

( )(
R
( La J me ) s+ La s + J me
a me
B
)
−1
θc ( s ) Ks A Ki n
=
θr ( s ) Ra B me s ( 1+ τ a s ) ( 1+τ me s ) + K b K i s+ K s A K ¿
La 0 J me 10
−3
Where τ a= = =0 , τ me = =2 × −3 =2 sec
Ra Ra B me 10

θc ( s ) Ks A K in
=
θr ( s ) Ra J me S2 +(K ¿ ¿ b K i + R a Bme )s + K s A K ¿ ¿

Hence ω n=±

Ks A K¿
Ra J me
K b K i+ R a Bme K b K i+ Ra B me
And δ = =2
2 Ra J me ω n √ K s A K i Ra J me n
26

θc 5A
On substitution of the numerical values we get = 2 …….. (18)
θr s 34.5 s +5 A
C(s) 5A
Example: Given Close Loop Transfer Function is M(s) = = 2
R(s) s 34.5 s +5 A
If A = 200, ω n=± √1000=± 31.6 rad /sec
δ=0.546
The Ch. Roots of the Ch. Eqn. s2 +34.5 s +1000=0 are:
s1 s 2=−17.25 ± j26.5 ; θ=tan-126.5/17.25=236.8° δ ωn t
For unit step response, i.e. θ r (t)=1 rad. the output θ c(t)

i. e. θc ( t )=L
−1
[[ 2
1000
s s +34.5 s+1000 ]]
=1+1. 2 e−17.25t sin ⁡(26.4 t + 236.8)

Comparison of Transient Response of 2nd order system with gain A varies as A=13.5, 200 and 1500

Gain A Damping Un- Max. % Delay Rise Time Settling Time for
ratio = δ damped Overshoot Overshoo Time = Td Tr Time = Ts Max
frequency t Overshoot
=ω n =tp
13.5 2.0 8.62 0 0 0.348 1.043 1.51 -
200 0.546 31.6 0.141 14.1 0.041 0.057 0.168 0.119
1500 0.2 86.2 0.52 52 0.012 0.014 0.15 0.037
27

VOL – 2, K3:

1. CONCEPT OF STABILITY

Basically, the design of feedback control systems can be regarded as problem of arranging the location of the
characteristic equation roots in such a way that the corresponding system will perform according to the prescribed
specifications. Among the many usual performance specifications, the most important requirement is that the
system must be stable at all times. Intuitively, the term stability is used to distinguish two classes of systems useful
and, useless. From a practical sense, a stable system may be useful one, whereas an unstable system is useless,
although there are a few exceptional cases.

Roughly speaking, stability in a system implies that small changes in the system input, in initial conditions or
in system parameters, do not result in large changes in system output.

A linear time – invariant system is stable if the following two notions of system stability are satisfied:

1) When the system is excited by a bounded input, the output is bounded. (BIOBO stability and also called
external stability)
2) In the absence of input, the output tends towards zero (the equilibrium state of the system) irrespective of
initial conditions. (This stability concept is known as asymptotic stability). This is also called internal stability
and very much important Non Linear Systems.

Let us observe the physical implication of the two notions of stability by considering a single – input, single –
output system with transfer function:

C (s ) b s m +b1 s m−1 +… … … … ..+bm


=G ( s )= 0 ; m< n------ (1)
R (s ) a0 s n +a1 s n−1+ … … ..+ am

With initial conditions assumed zero, the output of the system is give n by

c ( t ) =L−1 (G ( s ) R ( s ))=∫ ∞0 g ( τ ) r ( t−τ ) dτ

Where g ( t ) =∫ −1 ¿ is the impulse response of the System.

Taking the absolute value on both sides we get, |c ( t )∨¿| ∫ 0 g ( τ ) r ( t−τ ) dτ ∨¿


Since the absolute value of integral is not greater than the integral of the absolute value of the integrand,

|c (t )|≤ ∫ ¿ g ( τ ) r ( t−τ )∨dτ Endorse


0


≤∫ |g ( τ )|∨r ( t −τ )∨dτ ---------- (2)
0

The first notion of stability is satisfied if for every bounded input (|r(t)|≤M 1<∞), the output is bounded (Ic(t)I ≤
M2<∞). From eqn. 2, we have for bounded input, the bounded output condition as:


|c ( t )|≤ M 1∫ ¿ g ( τ )∨dτ ≤ M 2
0


Thus the first notion of stability is satisfied if the impulse response g(t) is integrated, i.e. , ∫ ¿ g ( τ )∨dτ is finite
0

(area under the absolute – value curve of impulse response g(t) evaluated from t=0, t o t=∞ must be finite).
28

The nature of g(t) is dependent on the poles of the transfer function G(s) which are the roots of the
characteristic equation. These roots may be both real and complex conjugate and may have multiplicity of
various orders. The nature of response terms contributed by all possible types of roots are given in Table 1 below
and illustrated in Fig.1

Table 1: Responds Terms contributed various types of roots.

Types of roots Nature of response terms contributed Illustrated figs


σt
i. Single roots =σ Ae Fig. 1(a)
k −1 st
ii. Roots of multiplicity k at s= σ ( A¿¿ 1+ A2 t + … … … .+ A k t ) e ¿
iii. Complex conjugate root pairs Aσte sin ⁡(ωt + β) Fig. 1 (b)
At σ = σ ±j ω
iv. Complex conjugate root pairs ¿
k−1 st
Of multiplicity k at s=σ± j ω … . …+ Ak t sin(ωt + β k )¿ e

v. Single complex conjugate root pair A sin (ωt + β ) Fig. 1 (c)


On the j ω axis (i.e. e at s ± j ω)

vi. Complex conjugate root pairs of A1 sin ( ωt + β 1 ) A 2 t sin ( ωt+ β 2 ) +¿ Fig. 1 (c)
k−1
Multiplicity k on the j ω – axis … . …+ A k t sin ( ωt+ β k )
vii. Single root at origin (i.e. at s = 0) A Fig. 1 (d)
viii. Double Roots at origin Fig. 1 (d)
2 k−1
ix. Roots of multiplicity k at origin ( A1 + A t+ … …..+ A k t ) Fig. 1 (d)

Certain observations are made from Table. 1. All the roots which have nonzero real parts [cases (i), (ii), (iii) and
(iv)], contribute response terms with a multiplying factor of e σ t . If s <0 (i.e. Roots have negative real parts), the
response terms vanish as t  ∞ and if s>0 i.e. σ>0 (i.e. the roots have positive real parts), the response terms
increase without bound. Roots on j ω – axis with multiplicity two or higher [cases (vi) and (viii)] also contribute
terms which increase without bound as t ∞. Single root at origin [case (vii)] or non-multiple root pairs [(case
(v)] on the j ω – axis contribute response terms which are constant amplitude or constant amplitude oscillation.
These observations lead us to the following general conclusion regarding system stability.
29

Roots in the s-plane corresponding impulse response

(a) Jω C (t)

x Stable
t

x c (t)

Unstable

(b)
jω c (t)
Stable
t


c (t)
Unstable

(c) c (t)
Stable

x Single pair of roots
x

c (t)

Jω Unstable
xx Double pair of roots

X
x

(d) Jω
Single root c (t) Stable
X σ
t

Jω Unstable
Double root
c (t)
X
X σ Fig: 1 Response terms contributed by various types of roots.
t
30

(1) If all the roots of the characteristic equation have negative real parts, then the impulse response is bounded

and eventually decreases to zero. Therefore, ∫ o ∨g ( τ ) ∨dτ is finite and the system is bounded input,
bounded output stable.
(2) If any root of the characteristic equation has a positive real part, g (t) is unbounded is infinite. The system is

therefore unstable. and ∫ o ∨g ( τ ) ∨dτ

(3) If the characteristic equation has repeated roots on the j ω - axis, g (t) is unbounded and ∫ o ∨g ( τ ) ∨dτ
infinite. The system is therefore unstable.
(4) If one or more non repeated roots of the characteristic equation are on the j-w axis then G(t) is unbounded

but ∫ o ∨g ( τ ) ∨dτ is infinite.The system is therefore unstable.

There are a few exceptions to the foregoing definition of– stability when a unit step input (R(s) =1/s) is
applied to a perfect integrator (G(s) = 1/s), the output is unbounded. However, an integrator is a useful system.
From Fig. 1 we observed that for non repeated poles of G(s) on the j w-axis, the response is bounded unless
input has a pole, matching one of the system poles on the j ω -axis i.e. in this case same as double i.e. repeated
pole. The zero input responses, in such case is bounded but non-asymptotic (C (t) does not tend to zero as (t
∞). generally referred to as the case Depending upon the amplitude of the ultimate response, such a system
may be treated as acceptable or non acceptable. This situation is of marginally or limitedly stable system.

In a vast majority of practical systems, the following statements on stability are quite useful:
(1) If all the roots of the characteristic equation have negative real parts, the system is stable.
(2) If any root of the characteristic equation has a positive real part or if there is a repeated root on the j ω -
axis, the system is unstable.
(3) If the condition (1) is satisfied except for the presence of one or more none repeated roots on the j ω -axis,
the system is limitedly or marginally stable.
In a further subdivision of concept of stability, a linear system is characterized as:
(i) Absolutely stable with respect to a parameter of the system if it is stable for all values of this
parameter i.e. a system is tested to determine whether stable or unstable i.e. the form of yes or no.
(ii) Conditionally stable with respect to a parameter, if the system is stable for only certain bounded
ranges of values of this parameter.

It follows from the above discussion that stability can be established by determining the roots of the
characteristic equation. Unfortunately, no general formula in algebraic form is available to determine the
roots of the characteristic equations of higher than second order.

However, simple graphical and algebraic criteria have been developed which, permit the study of
stability of a system without the need of actually determining the roots of its characteristic equation. These
criteria answer the question, whether a system be stable or not, in ‘yes’ or ‘no’ form.

Relative stability:

In practical systems, it is not sufficient to know that the system is stable but a stable system how
much stable it is and the degree of stability is a measure of relative stability. Parameters such as overshot
and damping ratio and settling time used in relation to the transient response provide indications of the
relative stability of a linear system in the time domain. In other words also, relative stability which is a
quantitative measure of how fast the transient die out in the system. The fastness is also indicated from the
location of roots whether away from the imaginary axis or close to the imaginary axis. If the roots are away
from the imaginary axis, the response will be faster. Gain – margin and Phase –Margin are the important
measure of relative stability in frequency domain.

There is nos. of methods of stability studies:


31

(1) Hurwitz stability criterion.


(2) Routh stability criterion.
(3) Routh – Hurwitz criterion
(4) Nyquist Criterion
(5) The Root Locus Plot
(6) Bode Diagram
(7) Lyapunov’s Stability Criterion etc.

2. Necessary Condition of Stability


Suppose that the characteristic equation of a linear system is written in the general form:
F(s) = a o s n + a1sn – 1+a2sn-2+………..+an-1s+an=0…….. (3)
Where all coefficients are real numbers:
In order that there are no roots of the last equation with positive real parts, it is
necessary but not sufficient that*:
[From the basic laws of algebra, the following relations are true for the given
polynomial (3):
a1 /a0 = -Σ all roots
a2 /a0 = Σ products of the roots taken two at a time
a3 /a0dτ = -Σ products of the roots taken three at a time
an /a0 = (-1)n products of all roots.
All these ratios must be positive and nonzero unless at least one of the roots has
positive real parts
* (1) All the coefficients of the polynomial have the same sign. Necessary Conditions
(2)None of the coefficients vanish

(a) Hurwitz Stability Criterion

For the stability of the system described by Eqn. (3), it is necessary and sufficient that
the n determinants formed from the coefficients a o, a1 --------a n of the characteristic
equation be positive, where these determinants are taken as the principal minors of
the following arrangement (called the Hurwitz determinant):

a1 a3 a5 a2n-1
a0 a2 a4 a2n-2
0 a1 a3 a2n-3
D n= 0 ao a2 a2n-4 = Hurwitz Determinant
0 0 a1 a2n-5
. . . .
…. . . .
0 0 0 an

| |
a1 a3 a5
D 1=a1> 0 ; D2=
| |
a1 a3
a 0 a2
>0 ; D3= a0 a2 a4 >0
0 a1 a 3

Here the coefficients with indexes larger than n or with negative indexes are replaced
by zero.
32

Hurwitz criterion is interims of determinants


(b) Routh’s stability criterion
Routh formulated an array:
e.g. F(s) = a 0 s6 + a1 s5 + a2 s 4 +a3 s 3 + a4 s2 +a5 s+a 6
s6 a0 a2 a4 a6
s5 a1 a3 a5 ¿

a1 a2−a3 a0 a a −a a a1 a6−0× a0
s4 = A 1 4 5 0 =B =a6 0
a1 a1 a1

A a 3−Ba1 A a5−a6 a1 A × 0−0 ×a1


s
3
=C =D =o 0
A A A

CB− AD C a6 −A ×0 C ×0− A ×0
s2 =E =a6 =o 0
C C C

ED ×C a 6
s1 =F 0 0 0
E

F a6 × E ×0
s0 =a6 0 0 0
F

The Routh stability criterion is stated as:

For a system to be stable, it is necessary and sufficient that each term of first
column of Routh array (as shown above) of if characteristic equation be positive if a 0>0. If
this condition is not met, the system is unstable and number of sign changes of the terms of
the 1st column of the Routh array corresponds to the number roots of the characteristic
equation in the right half of the s-place.

(c) Routh – Hurwitz Criterion


Routh criterion and Hurwitz criterion are equivalent: Elements of 1 st column of the
Routh array can be interpreted in term of Hurwitz determinants as:
6
s a 0=a 0

a 1=D1
Hence the condition of Positiveness of the Hurwitz determinants
s5
corresponds to the conditions of positiveness or no change of sign of
4
s A= D 2 ¿ D1 the 1st column of Routh array.
3 (1) All the coefficients of the polynomial have the same sign.
s C= D 3 / D2
(2) None of the coefficients vanish.
2
s E= D 4 ¿ D3
The two necessary conditions given above can be checked by
1
s F= D 5 ¿ D 4 inspection. However, they are not sufficient; it is quite possible that
s
0
a 6= D 6 ¿ D 5 a polynomial with all its coefficients positive and nonzero will have
roots in the right half of the s-plane. The necessary and sufficient
33

condition that all the roots of an nth order polynomial lie in the left half of the s-plane is
that the polynomial’s Hurwitz determinants D k (k = 1,2,3, ……………..,n) must be all
positive.

The Hurwitz determinants of Eqn. (7-22) are given by

| |
a1 a3 a5
D 1=a1
| |
a a
D1 1 3
a0 a2
D 3= a0 a2 a4
0 a1 a3

| |
a1 a3 a5 …. ….. a2 n−1
a0 a2 a4 … .. a2 n−2
0 a1 a3 … .. a2 n−3
D n=
0 a0 a2 a2 n−4
0 0 a1 … . a2 n−5
0 0 0 an

Where the coefficients with indexes larger that n or with negative indexes are replaced by
zeros.

Routh – Hurwitz’s criterion is stated as follows: The necessary and sufficient condition that
all the roots of the polynomial F(s) = 0, lie in the left half of the s-plane is that a 0>0, D1>0,
D1….. D n>0, where D1, D2 D3………….. D n are defined above

At first glance, the application of the criteria may seem to be formidable for high-order
polynomials because of the labor involved in evaluating the Hurwitz determinants.
However, the rule can be applied without actually working with the high order
determinants given above by following the steps given below.

The first step in the simplification of the Routh-Hurwitz criterion is to arrange the
polynomial coefficients into two rows. The first row consists of the first, third, fifth
coefficients, etc., and the second row consists of the second, fourth, sixth coefficients, etc.,
as shown in the following tabulation:
a 0 a 2 a 4 a6 a 8 ….
a 1 a3 a5 a7 a 9 ….

The next step is to form the following array of numbers obtained by the

s
6
a0 a2 a4 a6

s
5
a1 a3 a5 a7

a1 a2−a3 a0 a1 a2−a0 a5 a1 a6−a0 ×0


s
4
a1
=A
a1
=B
a1
=a6 0
34

Aa3−a1 B Aa5−a1 a6 A × 0−a1 ×0


s
3
=C =D =0 0
A A A

CB−AD Ca6 − A ×0 C ×0− A ×0


s
2
C
=E =a6
C
=0 0
C

ED−C a 6
s1 =F 0 0 0
E

F a6 −E ×0
s
0
=a6 0 0 0
F

The last step is to investigate the signs of the numbers in the first column in the last
tabulation. The roots of the polynomial are all in the left half of the s-plane if all the
elements of the first column are of the same sign if there are changes of signs in the
elements of the first column, the number of sign changes indicates the number of roots
with positive real parts. The reason for this statement is apparent, since the relations
between the elements in the first column and the Hurwitz determinants are given as
follows:

s
6
a 0=a 0

s
5
a 1=D1 Hence the condition of postiveness of the Hurwitz
Determinant corresponds to the condition of
s4 A=D2 /D 1
postiveness of elements of the first column of the
s
3
C=D3 / D2 Routh Array.

s2 F=D4 /D 3

s
1
F=D5 / D 4

s
0
a 6=D6 / D 5

Example -1 Consider the polynomial: (s - 2)(s + 1) (s – 3) = s 3 – 4s2 - 5s + 6 = 0

The polynomial given in the last equation has negative coefficients; thus, from the necessary condition, we know
without applying Routh’s test that there are roots with positive real parts. But for the purpose of illustrating Routh-
Hurwitz’s criterion, the Rouths tabulation is formed as follows:

Change in sign s
3
1 -5

s
2
-6
(−4 )(−5 )−6
Change in sign s
1
=−3.5 0
−4

(−3.5 ) ( 6 )−(−4 ) ( 0)
s0 =6
−3.5
35

Since there are two sign changes in the first column, the polynomial has two roots located
in the right half of the s-plane. This checks with the known result, since it is know that the
two unstable roots are s=2 and s=3

Example -2 consider the polynomial

2 s 4 + s3 +3 s2 +5 s +10=0

Since the polynomial has no missing terms, and the coefficients are of the same sign, it
satisfies the necessary condition of stability. However, it is still necessary to check the
sufficient condition. The Routh’s tabulation is still necessary to check the sufficient
condition. The Routh’s tabulation is

s
4
2 3 10

s
3
1 5 0
( 1 )( 3 ) −( 2 ) (5)
Change in sign 10 0
2
s =−7
1

(−7 ) ( 5 )−( 1 ) (10)


Change in sign s1
−7
=6.43 0 0

s
0
10

Since there are two changes in sign in the first column, the polynomial has tow roots with
positive real parts.

Special Cases:

Occasionally, in applying Rout-Hurwitz’s criterion, the following kinds of difficulties may


occur:

(1) The elements in any one row of the Routh’s tabulation are zero, but the other
elements are not.
(2) The elements in one row of the Routh’s tabulation are all zero. These two cases will
be discussed separately as follows:
(1) When the first element in any row of the Routh’s tabulation is zero, but the other
elements are not: if a zero appears in the first position of a row, the elements in the
next row become infinite, and Routh’s test breaks down. To restore the missing
power of s, simply multiply the polynomial by the factor (s+a) where a is any positive
real number, * and carry on the usual Routh’s test.

Example 3 Consider the equation

( s−1 )2 ( s +2 )=s3 −3 s +2=0 ⋯ ⋯( s .1)


36

Since the coefficient of the s2 term is zero, we know from the necessary condition that there
must be at least one root of the polynomial which is located in the right half of the s-plane.
The Rouths tabulation of the coefficients is given below.

*Although a negative real number is also allowed, it contributes roots in the right half of the
plane, and this root should be taken into account when one is interpreting Rouths
tabulation.

s
3
1 -3

s
2
0 2

s
1

0
s

Because of the zero in the first element of the second row, the first element of the third
row is infinite. To correct this situation, simply multiply the Eqn. (s. 1) by the factor (s + a).
If the constant ‘a’ is arbitrarily chosen as 3 (for reasons which will become apparent later,
the value of a is not chosen as 1 or 2), we have

( s−1 )2 ( s +2 ) ( s+ 3 )=s 4 +3 s 3−3 a2 −7 s+ 6=0


The rouths tabulation of the last equation is
s
4
1 -3
s3 3 -7
−9+7 −2
Change in sign 3
=
3
6
s2
Change in sign

s
1
( −23 ) (−7) −18 =20 0
( −23 )
s0 6
Since there are two changes in sign in the first column of the Rouths tabulation, two roots
of the equation are in the right half of the s-plane. As an alternative to the remedy of the
situation described above, we can replace the zero element in the Rouths tabulation by an
arbitrary small positive number Є, and then proceed with the Routh’s test. For instance, for
the equation given in Eq. of the example if we replace the zero elements in the second row
of the Rouths tabulation, and proceeded with the tabulation, we have.

s
3
1 -3

s
2
Є 2
−3 Є−2
s
1
=−ve 0
Є
37

s0 2

Now since (-3e -2)/Є approaches -2/e, which is a negative number, the first column of the
last tabulation has two changes in sign. This agrees with the result obtained earlier.

(2) When all the elements in one row of the Routh’s tabulation are zero: This condition
indicates that there are pairs of real roots with opposite signs, pairs of conjugate roots on
the imaginary axis, or both; or conjugate roots forming a quadrate in the s-plane. The
equation corresponding to the coefficients just above the row of zeros is called the auxiliary
equation. The order of the auxiliary equation is always even, and it indicates the number of
the root pairs that are equal in magnitude but opposite in sign. For example, if the auxiliary
equation is of the second order, there are two equal and opposite roots. For a fourth order
auxiliary equation there must be two pairs of equal and opposite roots. All these roots with
equal magnitude can be obtained by solving the auxiliary equation. Again, Routh’s test
breaks down; in this case, because of the row of zeros. To correct this situation, simply take
the first derivative of the auxiliary equation with respect to s; replace the row of zeros with
the coefficients of the resultant equation obtained by taking the derivative of the auxiliary
equation, and carry on with the Routh’s test.
Example -4

Consider the same equation used in Example 3, s3 – 3s +2=0. In multiplying this equation by
a factor (s + a), logically the first number that comes into one’s mind would be a=1. Thus,

(s -1)2 (s+2)(s+1)=s4+s3 – 3s2 –s + 2=0

The Routh tabulation of this equation is


s
4
1 -3 2

s
3
1 -1 0
−3+1
s
2
1
=−2 2 coefficient of the auxiliary equation
2−2
s1 =0 0
−2
Since the s1 row contains all zeros, Routh’s test breaks down. The multiplication of the
factor (s+1) to the original equations, which has a root at equation is obtained by using the
elements contained in the s2 row as the coefficients of the equation. Thus,

A(s) = 2s2 + 2=0

And d A(s)/d s = -4s

Now the row of zeros in the Routh’s tabulation is replaced by the coefficients of the last
equation; the Rouths tabulation reads as follows:
38

s4 1 -3 2

s
3
1 -1

Change in sign s2 -2 2

s1 -4 0 (coefficient of d A(s)/d s)
Change in sign s0 2 0

Since there are two changes in sign in the elements in the first column of the new Routh’s
tabulation, two roots of the equation have positive real parts. By solving the roots of the
auxiliary equation in Eq. (7-29) we have
S2 = 1, or s = ±1.
Example .5 Consider the equation
(s+2)(s - 2)(s + j)(s - j)(s2 + s + 1)= s6 + s5 - 2s4 – 3s3 – 7s2 - 4s + 4
It is known that the last equation has two pairs of equal roots with opposite signs at s = ±2
and s = ±j. The Routh’s tabulation is

s
6
1 -2 -7 4

s5 1 -3 -4
−2+3 −7+ 4
s
4
1
=1
1
=−3 -4

s3 0 0 0

The auxiliary equation is

A(s) = s4 – 3s2 – 4 = 0

Which indicates that there are two pairs of equal roots with opposite signs: The first
derivative of the auxiliary equation with respect to s is

d A(s )/d s = 4s3 – 6s

From which the coefficients 4 and 6 – are substituted into the row of zeros in the Routh’s
tabulation. The new Routh’s tabulation is

s
6
1 -2 -7

s
5
1 -3 -4

s
4
1 -3 -4

s
3
4 -6 0 Coefficients of d A(s)/d s
39

−12+ 6
s2
4
=1.5 -4 0
−9+16
s
1
−1.5
=−16.7 0
s0 -4 0
Since there is only one change in sign in the first column of the new Routh’s tabulation, the
polynomial has one root with a positive real part. This result obviously checks with the
given polynomial roots. The two pairs of equal roots are obtained by solving the auxiliary
equation given in above AE. The roots are s = ± 2 and s = ± j.
Example 6: A Frequent use of Routh’s criterion is to determine the condition of stability of
a linear feedback control system. For instance, the servo system with integral control
shown in Fig. has the characteristic equation s3 + 34.5s2 + 7500s +7500K1 = 0
The Routh’s criterion is to be applied to the last equation to determine the range of value
of K1 for which the closed – loop system is stable. The Routh’s tabulation of above Eq. is
s
3
1

all 7500

s
2
34.5 7500K1
s
1
(258,750−7500 K 1 )/34.5 0
0
s 7500K1
For the system to be stable, the coefficients in the first column of the Routh’s tabulation
must be positive. The conditions are.
(258,750 – 7500K1)/34.5>0
And 7500K1 > 0
From the above condition, we have
K1<34.5
And from the other condition, we have
K1>0
Hence, the condition for stability is that K1 must satisfy the relation 0<K 1<34.5
Example 7: Consider the characteristic equation of certain closed loop system,
3 2
s + Ks + ( K +2 ) s +4=0
The Routh’s tabulation is
s
3
1 (K+2)

s
2
3K 4
3 K ( K +2 ) −4
s1 0
3K
s0 4

From the s2 row, the condition of stability is K>0


And from the s1 row, we have
3K2 +6K – 4 > 0
40

−6 ± √ 36+48
is k= From which K < -2.528 or K > 0.528
6
Is k =−1 ± √ 2.35
When the conditions K > 0 and K > 0.528 are compared, apparently the latter
limitation is the most stringent one. Hence, for the closed –loop system to be stable the
value of K must be greater than 0.528.

Limitations of Routh- Hurwitz Criteria:

(1) It should be reiterated that the Routh-Hurwitz criterion is valid only if the
characteristic equation is algebraic and that all the coefficients are real. If any one of the
coefficients of the characteristic equation is a complex number, or if the equation contains
exponential functions of s, such as in the case of a system with time delays, the Routh-
Hurwitz criterion breaks down completely.

(2) Another limitation of the Routh –Hurwitz criterion is that it offers information only on
the absolute stability of the system. If a control system is found to be stable by the Routh’s
test, one still does not know how good the system is – in other words, how closely the roots
of the characteristic equation are located to the imaginary axis of the s-plane. On the other
hand, if the system is unstable, the Routh’s test gives no indication on how the system can
be stabilized. For information of the relative stability of a control system, one must turn to
the Nyquist criterion or the root locus method.

The Nyquist Criterion:

Thus far, two methods of determining the location of the roots of the characteristic
equation have been indicated:

(1) The roots are actually determined by solving the characteristic equation.
(2) The location of the roots with respect to the imaginary axis of the s-plane is
determined by means of the Routh-Hurwitz criterion.
Although the application of the Routh-Hurwitz criterion is quite straight forward, the
only information that the criterion can furnish is the absolute stability of the system.
The Nyquist criterion possesses the following features which make it particularly
desirable for the stability analysis of feedback control systems:
(1) It provides the same amount of information on the absolute stability of feedback
system as the Routh-Hurwitz criterion.
(2) In addition to the absolute system stability, the Nyquist criterion also indicates the
degree of stability of a stable system, and gives information on how the system
stability may be improved, if necessary.
(3) The Nyquist locus gives information concerning the frequency response of the
system.
41

Consider the feedback control system shown in Fig. 7-3. The closed-loop transfer
C ( s) G (s )
function of the system is =
R (s ) 1+G ( s ) H ( s)

(The details of Nyquist method will be discus letter)

Relative stability information from Routh Criterion:

z - Plane axis (shifted origin) s- Plane axis


jω jω jω jω

01 01 0
σ σ
Example: 0 1
Ch. Eqn. s3 + 7s2σ+1 25s + 39 =0
Shifting the origin: s=z – σ 1, =z – 1 (Taking σ1 = 1)
Hence w. r. t o1 origin, the Ch. Eqn. becomes =
(z - 1)3 + 7(z – 1)2 + 25 (z – 1) +39 =0
Or (z2 + 1 – 2z) (z – 1) + 7(z – 1)2 + 25z – 25 +39 =0
Or (z2 + 1 -2z) (z-1+7) +25z +14 =0
Or z3 +z -2z2 + 6z2 + 6 – 12z + 25z 14 =0
Or z3 +4z2 + 14z + 20 = 0
Routh array
z3 1 14
2
z 4 20
4 × 14 – 1 ×20
z1 4
=9
9 ×20 – 4 ×0
z0 9
= 20
No sign change in the 1st column which implies the roots of original ch.eqn in s-domain lie
to the left of z (i.e. s = -1 axis) plane.
Example-8:
Head nod
Sensed

Fig: 1
The block diagram of Fig.1 above is representative of wheel chair velocity control initiated
by head nod of a paralyzed patient. Of course of the wheel chair to move from one position
in XY-plane to another, an outer position control loop would be needed; this is not being
considered here.
(a) Determine the limiting value of forward gain K(=K1K2K3)for the system to be stable if
τ1 = 0.2s, τ2 = 1 s and τ3 = 0.4s
42

(b) Calculate the gain for the system to be stable with a settling time of 4s based on
dominant poles.
(c) With K as in part (b) find all the roots of the ch. Eqn.
Solution:
a. The systems ch. Eqn. can be written as (0.2s + 1)(s + 1)(0.4s + 1)+K=0  1+G
K1 K2 K3
Where G =
(τ¿ ¿1 s+1)(τ ¿¿ 2 s +1)(τ ¿¿ 3 s +1) ¿ ¿ ¿
Or s3 + 8.5s2 + 20s + (12.5) (1 + K) = 0 ………… (i)
Applying Routh Criterion s3 1 20
s2 8.5 12.5 (1+K)
s 18.5 ×20 – 12.5(1+ K ) x 1
8.5

For all terms of the 1st column to be positive (same sign) 8.5x20 – 12.5 (1+K) >0 or K<12.6 or
Limiting value of K = 12.6

b. Settling time, t s = 4/δ ω n or δ ω n = 4/4 = 1

Thus the real part of dominant root (or complex root pair should be -1 or more
(i.e. ω δ n =-1)). To ascertain this and to find the corresponding value of K 1, we get s=z – 1

Hence the Ch. Eqn. becomes (z – 1)3 + 8.5 (z – 1)2 +20 (z – 1) +12.5 (1+K)=0

Or z3 +5.5z2 + 6z + 12.5K = 0, i.e. the roots are shifted from origin towards left by1

Applying Routh’s Criterion z3 1 6


z2 5.5 12.5K: A.E
5.5 x 6 – 1 x 12.5 K
z1 5.5
z0 12.5K
For the limiting gain of the dominant roots of characteristic eqn. to be -1 is given as 6x5.5K
– 12.5K = 0 or K=2.64. For this gain one can find the imaginary part of the dominant roots
from the auxiliary eqn. 5.5s2 + 12.5 x2.64 = 0 or s = ± j2.45
(c ) From part (b) δ ω n = 1, ωn2 = (1)2 + (2.45)2= 7 jw
The dominant roots are given by the factor (s2+2δωns + ωn2) = s2+2s+7
ωn
The third root is obtained by dividing out the Ch. Eqn. (i) (with K=2.64),
by the factor, s2+2s+7. The Ch. Eqn. is s3 + 8.5s2 + 20s + 12.5(1+K) = 0 X 2.45j
Or s3 + 8.5s2 +20s +45.5 = 0
Hence
δ ωn 0 σ

ω=-1
n

X -2.45j
2
s +2 s +7
¿

Thus the roots of the Ch. Eqn. (if K=2.64) are s1, s2 = 1 ± j 2.45 and s3 = -6.5 (Ans.)
43

VOL – 2 – K4
1. Root Locus:
The root locus concept: (Variation of open loop forward Gain K effects on the location of
poles in S-plane)
Root locus method is the graphical method for obtaining the roots of the system
characteristic equation in s-plane and thereby investigating the system response
performance. This method was devised by W.R. Evans in 1948 and clearly indicates the
effects of system performance in time and frequency domains.
M(s) = C(s)/R(s) = G(s)/1+G(s) H (s)……………… (1)
The Ch. Eqn. of the system is 1+G(s) H (s) = 0…………………… (2)
Eqn. (2) contains a transportation lag term 1 and a rational algebraic function G(s) H (s). The
diagram containing the locus of the roots of the Ch. Eqn. (2) when K (the open loop Gain)
is varied from 0 to ∞ is known as the Root Locus of the system.

Eqn. (2) can be written as F(s) = G (s) H(s) = - 1 ……………… (3)

In the polar form |F (s )|=|G ( s ) H (s )|=−1………………… (4)

From Eqn. (4), the necessary conditions for the point to be present on the root locus are:

¿ F (s)∨¿|G(s ) H (s)|=+1 ……………… (5)

………… (6)

Where k is an integer defined by k = 0, ±1, ±2………….

The Ch. Eqn.(2) can be written in the form of:


K ( s + z 1 )( s+ z2 ) … … … (s + z n)
G ( s ) H ( s )= =−1 ……………… (7)
( s+ p1 ) ( s+ p2 ) … … …(s+ p m)

( s+ z 1 ) ( s + z 2 ) … … …(s+ z n) −1
¿ = ⋯ ⋯ ⋯(7 ')
( s + p1 )( s+ p 2 ) … … …(s+ pm ) K

From Eqn. (5), (6) and (7), the magnitude and phase angle requirement for the points on
the root locus are:

1

44 θP1 A
F C
E θP
n B
K
∏i=1
|s+ z i| S -Plane
X o σб
-z1 θP θP θP
|F (s )|= ………………… (8)
n
∏ j=1
|s + p j| 2 3 3

n m
∑ ( 2 k +1 )− ∑ =(2 k +1) …………………. (9)
i=1 j=1
K ( s + z 1 )( s+ z2 )
E.g. F ( s ) = i.e. n=2, m=4, D=s,
s ( s+ p1 ) ( s+ p2 ) ( s+ p3 )
P2 P3

K|( s1 + z 1)||(s 1+ z 2)| Fig. 1: Pole zero plot of F(s)


|F (s )|= =+1 ………. (10)
s 1|( s 1 p1 )||( s 1 p2 )||( s1 p 3 )|

…… (11)

From Eqn. (10) and Eqn. (11) and Fig: (1), the necessary condition for the point s 1 to be on
AB 1
the root locus in terms of vectors CDEF = K ….. (12)
And θz1 + θz2 – (θp0+θp1 + θp2 + θp3) = (2k + 1) π ……… (13)
Relation between the poles and zeros of the Loop Transfer Functions and the Ch. Eqn.
Roots: Needed for construction of roots locus
Rules for construction of Root Loci:
(1) The root locus starts from the poles of G(s)H(s)
n
∏ |s + z i|
i=1 1
Proof: |F (s )|= m = …… (14) = (7’)
K
∏ |s+ p j|
j=1
For K=0, that is at the starting point of the root locus the right hand side of Eqn. (14) is
infinity for which s approaches to the poles of the Loop transfer function G(s) H(s). Hence
the locus of the roots of Ch. Eqn. starts from the poles of the loops transfer function
G(s) H (s).
(2) The root locus terminates at zeros of G(s)H(s)
Proof: For K=∞ i.e. at the terminating points of the root locus the right hand side of Eqn.
(14)is 0 for which s approaches to the 0 of the loop transfer function G(s)H(s). Hence the
locus of the Ch. Eqn. roots terminates at the zero of the loop transfer function G(s) H (s).
(3) Number of Separate Root Locus is equal to the number of finite Poles or Zeros whichever is
greater.
Proof: Since the root locus starts from poles of G(s) H (s) and terminates at the zeros of
G(s)H(s), the number of separate root locus N is obtained from
N=Z for Z>P
=P for P>z ------- (15)
(4) Root locus and the inverse Root Locus (i.e. for K= - ∞ to zero) are symmetrical w. r. t the
real axis:
45

Proof: Generally the Loop Transfer Functioning of Linear Control System is rational
functions for which complex roots of Ch. Eqn. appear in conjugate pairs. Hence the root
locus and the inverse root locus are always symmetrical w r t the real axis.
(5) The root locus and the Inverse Root Locus are asymptotic straight lines with specified
( 2 k +1 ) π
angles for large values of s. the angles of asymptotes are defined by: θ= (16) for
P−Z
Root locus
2k π
θ= ------- (17) For Inverse Root Locus
P−Z
Where k=0, 1, 2 ---- up to k=(p-z)= positive integer exclusive.
Proof: The Loop Transfer function can also be written as,
K ( s n+ a1 s n−1+ … … … … ..a n )
H ( s ) G ( s )= =−1---------- (18)
[ sm +n +b1 s m+ n−1 +… … …..+b m+n ]
For convenience the no of the poles taken = m + n i.e. the difference of poles and zeros = m
K
¿
[s m+n
+b1 s
m +n−1
+ … … … .b m+n ]
[ sn +a 1 sn−1 +… … … .+a n ]
K
¿
m m−1 R (s ) ……. (19)
s + ( b1 −a1 ) s +… … … .+
P (s)
Where R(s) is a polynomial having degree of s less than n and
P(s) = s n + a1 sn-1 + ……. an ……. (20)
From Eqn. (19), s m + (b1 – a1) s m-1 + ……… + R(s)/P(s) = -K ------- (21)
For larger values of s in Eqn. (21), the term R(s)/P(s) approaches zero and only first two
terms are predominating. Hence
sm + ( b1−a 1) sm −1 =−K

Or sm +( b −as ) s =−K
1 1 m

Or s [ 1+(
s )]
b −a
m 1 1
=−K

[ ]
1 1
b −a m
Or s 1+ 1 1 =[ −K ] ……….. (22)
m
s
Expanding Eqn. (22)

[ ]
1
b1−a1
+… … …¿ = [−K ] ………. (23)
m
s 1+
ms
Neglecting higher order terms in Eqn. (23), we get,
1
b1−a1
s+ =(−K ) ………. (24)
m
m
Substituting s=σ +jω in (24)
σ + jω+
b1−a 1
m
1
m
≃ K cos
m
| |[
( 2 k +1 ) π
+ jsin
( 2 k +1 ) π
m
……… (25) ]
For 0<k<∞ [ ∵−1=cos ⁡(2 k +1) ] π + jsin(2 k +1) π
| |[ ]
1
b1−a 1 2K π 2k π
And σ + jω+ m
≃ K cos + jsin for−∞ <k < 0……….. (26)
m m m
46

Where k=0, ± 1, ± 2…….


Equating real and imaginary parts of Eqn. (25) separately
σ+
b 1−a1
m
| |
1
m
≃ K cos
(2 k +1) π
m
| |
1
And ω= K m sin
( 2 k +1 ) π
m
………….. (27)

b1−a1
σ+
| |
1
m ω m
From (27) K = (2 k +1) π = (
2 k +1 ) π
sin ⁡ cos
m m
Hence ω=tan
( 2 k +1 ) π
m
σ+[ b1 −a
m
1
]
……….. (28)
Eqn. (28) represents a straight line in s-plane by
ω = tan θ (σ - σ ₁) -------- (29)
( 2 k +1 ) π ( 2 k +1 ) π
Where tan θ = slope of the straight line ¿ tan =tan
m P−Z
( 2 k +1 ) π
Or θ= …………… (30)
P−Z
−b1−a1
And σ 1 = Intercept on the real axis = ……… (31)
P−Z
Where k=0, ± 1, ± 2, ⋯ ⋯ ± K = (P – Z) (Exclusive).
Expressions (30) and (31) define the angles and the real axis intercepts for 0<K<∞ i.e. for
root locus. Similarly from Eqn.(26), the angles and real axis intercepts for inverse root locus
(- ∞ < K <0) are defined as:

2k π (b ¿ ¿1−a1 )
θ= ; σ 1= ¿……… (32)
P−Z P−Z
(6) For the function G(s) H(s) having P finite poles and Z – finite zeros the number of
asymptotes is m = P – Z. These asymptotes intersect on the real axis in s - plane at a point
=−{ }
∑ Poles of G ( s ) H ( s ) −∑ zeros of G ( s ) H (s)
defined by: σ 1=− ( b −a
1
m ) 1
P−Z
Where Σ poles of G(s) H(s) = sum of the distances of G(s) H(s) poles from the origin =b 1 and
Σ zeros of G(s) H(s) = sum of the distances of G(s) H(s) zeros from the origin =a 1
σ 1 is called Centroid.
E.g. 1 Determine the number of asymptotes for the Loop Transfer Function
K (s +1)
G(s) H (s)= . Hence calculate the angles and intercept of asymptotes with
s ( s+2 ) ( s +3 ) (s+ 4)
the real axis, when K is varied from 0 to ∞.
Solution: P=4, Z=1 i.e. p1 = 0; p2 = - 2; p3 = - 3; p4= - 4; and z1 = -1
(i) Root locus starts from s=0, -2, -3 and -4 and terminates at s =-1 and at three other zeros
at ∞
(ii) Number of asymptotes = m = P-Z= 4-1 =3 Jω
S-plane
(iii) The angle of asymptotes from Eqn. (30):
( 2 k +1 ) π 180 ° 180° Asymptotes
θ= , θ1= 3 =60 ° for k =0
P−Z 60°
47

3 ×180 -4 -3 -2-1 0 2
θ2 = =180 ° for k=1 σA=π r
3
5 ×180 300°
θ3 = =300 ° for k=2
3
[ 0−2−3−4 ] −[ −1 ] −8
σ 1= = =−2.666
3 3
K
E.g. 2 A unit feedback systems has G ( s )=
s ( s+ 4 ) ( s2 +2 s +5 )
(i) Calculate number of finite poles and zeros. Hence obtain the number of zeros at ∞
(ii) Number of separate root loci?
(iii) The angle of asymptotes and centroid in the pole – zero plots.
K
G ( s )=
s ( s+ 4 )( s+1+2 j ) (s+1−2 j)
(i) No. of zeros at ∞ = P-Z = 4 – 0 = 4
(ii) No. of separate root loci ¿ P=4 ∵ P> Z
(iii) No. of asymptotes = P-Z =4-0 = 4=m
( 2 k +1 ) π ( 2 k +1 ) π
(iv) θ= ; θ1 = =45 ° for k =0
P−z 4
[ 0−4−1+2 j−1−2 j ] −0 ( 2 k +1 ) π
σ 1= θ2 = =135° for k=1
4 4
(2 k +1) π
=-1.5 θ3 = =225 ° for k =2
4
(2 k +1) π
θ 4= =315 ° for k=3
4
(7) Real Axis Locus
For positive values of K (i.e. the root locus) the point of the roots locus on the real axis lie to
the left of an odd number of finite poles and zeros of G(s) H(s).
(For negative values of K (i.e. for inverse root locus) the points of inverse root locus on the
real axis lie to the left of an Even number of finite poles and zeros of G(s)H(s)
Proof:
θ1

s1
Fig.1 Fig:
θ1

The Complex conjugate poles of G(s) H (s) shown above. Fig. shows that angles of vectors
formed by joining these poles to s1 on the real axis are cancelled because these angles are
equal in magnitude but opposite in direction. Hence the complex conjugate poles or zeros
of G(s) H(s) contribute no angles at the root locus or inverse root locus. The only
contribution of angles on the real axis is made by the poles and zeros which are located on
the real axis.

F F
Zero degree
i js-plane
ω 180° js-plane
ω
i
g g
s1 . б s1 s1 . б
a c
48

Fig: a Fig: c
The poles and zeros which lie on the real axis to the left of the specified section of real axis
contribute zero angle (shown in Fig. (a) (b)) while each pole and each zero to the right of
the section contribute 180° jω

s-plane
180°
Zero degree S-plane
S1 s1 б
s б Fig. d
Fig. 2(a), (b), (c) & (d)
Fig.b
(b
Therefore the total angle due to poles PR and zeros ZR located on the real axis to the right of
the specified section contributes (PR +ZR) π degree to the locus on the real axis. From Eqn.
(11) the condition for the point to lie on the root locus is

π ¿(P R + Z R ) π

Hence for the total number of poles and zeros of the right of the specified section is
odd multiple of π, and for these sections of root locus lie on the real axis. Similarly for
inverse root locus for the total number of poles and zeros to the right of the specified
section is Even multiple of π and for these sections of the inverse root locus lie on
the real axis.
Eg.3: Sketch the sections of real axis in s-plane which lie on the root locus for the loop
K ( s+ 1 ) (s +3)
transfer function: G ( s ) H ( s )= jω
( s+2 ) ( s +4 )( s+5 )( s+6 ) ( s 2+2 s+10 )
K ( s+1 ) ( s+ 3)
Solutions: G ( s ) H ( s )= ( s+2 ) ( s +4 )( s+5 )( s+6 )( s+1+ j3 )(s+ 1− jω) +j3
+j2s-plane
The selections AB, CD and EF will lie on the root locus
+j1
F E D C FB A б
-6 -5 -4 -3 -2 -1 -j1
-j2

(8) Angle of Departure and Arrival -j3


The angles of departure and arrival of root locus at the complex poles and zeros
respectively depend upon the contribution of angles due to other poles and zeros of the
loop transfer function G(s) H (s). The departure and arrival angles θ d and θ a are derived as
follows:

p4
49

¿ ¿) θd
z3

θz 3
S-plane
θz n
θp3 θz 1 θp1
zn σ
p3 z1 p1 σ
Pm+n
θp2
θz 2 p2
z2 Fig. a

Considering the Fig: (a) using the condition of Eqn. (6), we get
(θz1+θz2+θz3+………..θ z n) – (θd+θp1+θp2+θp3+…………..+θ p m + n) = (2k+1) π
or θ d = Angle of departure of the root locus at the complex pole
= [(2k+1) π+ (θp1+ θp2+ θp3+……………..θ pm +n)-(θz1 + θz2+ …………. θz3)¿
z3 z a ( z 3 ¿ θp4 p4 jω

S-plane

zn σ
θPm+n
z2 Fig: b

Similarly considering the fig (b) above and using the Eqn.6 we get,
(θa+θz1+θz2+………..θzn) – (θp1+θp2+θp3+…………..+θ p m+n) = (2k+1) π
θ a = Angle of arrival of the root locus at the complex zero at Za
=[(2k+1) π+(θp1+ θp2+……………..θ p m+n)-( θz1 + θz2+ ………….+ θzn)
E.g. 4 Sketch the pole zero configuration for the unity feedback system with
2
K (s + 8 s+ 41)
G ( s )= . Hence determine the angle of departure at the pole
s (s+2) ( s +2 s+101 )
2

s=-1+j10 and the angle of arrival at zeros = -4+j5


K ( s2 +8 s+ 41 ) k ( s+ 4+ j5 ) (s +4− j5)
Solution: Given G ( s ) = 2
=
( )( )
∧H ( s )=1
s ( s+ 2 )( s + 2 s+101) s s+2 s +1+ j 10 (s+1− j10)

j10
j 10
+j5
S-plane

-
3
-4 f3 -2
f 2
f
50

z2

4
θ p1=90+ tan −1 =90+38.7
−1
θp1=90+tan 0.1=95.8 ° 5

3
−1
θp2=90° θp2=90+tan =90+1.13
15
10
−1
θp3=tan =8.43° 2
1 θp3=90+ tan−1 =90+21.8
−1 5
5
−1 5
θz 1=tan =5.90 ° θp4 =180+ tan =180+59.5
3 3
15
θz 2=tan−1 =7.87 ° θz 2=90 °
3

∴ θd=−[ 180+ ( θp1 +θp2 +θp3 ) −( θz 1+θz 2 ) ] θ a=[ 180 °+ ( θp1 +θp2 +θp3 )−θz 2 ]
¿−[ 360+ 4.6 ] =−4.6 ¿ 121.13 °

9. Interaction of the Root Locus with Imaginary Axis:


Example: Characteristic Equation: s3 +bs 2+ cs+ Kd=0
Routh Array:
s3 1 c
2
s b Kd …..(AE)
1
s (bc - Kd)/b 0
0
s Kd 0
Equating the first element in the row corresponding to s1 equal to zero,
bc−Kd
b
bc
=0→ K= ;From AE: bs2 + Kd=0 → s= Kd = bc d =√ c
d b d b √ √
Hence the intersections of the root locus with the imaginary axis conditions are:
bc
s= √ c∧K=  The value of K at this point is known as static sensitivity of the system.
d
E.g. 5: Calculate the value of K and s for the point in s-plane at which the root locus of
K
G ( s ) H ( s )= 2
; intersect the imaginary axis.
s ( s +2 ) (s +2 s+ 2)
Solution: Ch. Eqn. is 1+G(s) H (s) =s(s+2) (s2+2s+2) +K=0=s4+4s3+6s2+4s+K

s4 1 6 K
s3 4 4 0
4 × 6−1 × 4 4 × K−1 ×0
s2 4
=5
4
=K 0
5× 4−4 × K
s1 5
=0 0 0
s0 K 0 0

20−4 K
Hence =0 → K =5 ; Auxilary Eqn. (A.E) is 5s2+K=0 Or s = ±ji
K
51

Hence the points at which the root locus intersects the Imaginary Axis are K=5, s=±j
10. Breakaway points
The points on the s-plane at which two or more branches of root locus depart or
arrive at are known as breakaway points or saddle points.
Properties of Breakaway Points: It indicates double or multiple roots at the breakaway
points.
(i) Breakaway point represents the multiplicity of the roots of characteristic Eqn.
(ii) Breakaway points either lie on real axis or occur in complex conjugate pairs.
(iii) The root loci or inverse root loci approach or leave the breakaway point on the real axis
at angle of 180/n where ‘n’ is the number of root loci or inverse root loci which are
approaching or leaving the breaking point.
(i) The roots at the breakaway points of root loci are obtained by using any one of the
procedures described below.
Procedure – I
dG ( s ) H ( s)
=0 → Solution of this equation gives the breakaway points on the root locus of
ds
the loop transfer function G(s) H (s).
Proof: The Ch. Eqn. is 1+G(s)H(s) = 0 ---------- (1)
P(s) P( s)
Or 1+ K Q(s) =0 --- ------ ---- ---------- (2) (i. e putting G(s) H (s)=K Q(s ) )
Or Q(s) +K p (s) = 0 ------ ----- ----- ---------- (3)
A small increment ΔK in the parameter K changes (3) into
Q(s) + (K+ΔK) P (s) = 0 --------------- ---------- (4)
Dividing Eqn. (4) by [Q(s) +KP(s)], we get,
ΔKP (s)
1+ ------------ ----------- (5)
[Q ( s )+ KP ( s ) ]
Or 1+ΔKB(s) = 0 ------------ (6)
P( s)
Or 1+∆ KB ( s )=0 where B ( s )= ----------- (7)
[Q ( s ) + KP ( s ) ]
The denominator of B(s) in Eqn. (7) is the left hand side Ch. Eqn. (3)
At the points which are very close to the characteristic eqn. roots s i of multiplicity n which
is the breakaway point of n loci, function B(s) is approximately:
A1 A1 ∆ K Ai
B ( s )= n
= n
; Here Ai is constant. From Eqn. (6) we can write, 1+ n
=0 or
(s−s i) ( Δs i) (∆ si )
n−1
∆ K Ai
(∆ si )
n
=−1 or Δ K =
Δ si Ai( )
( Δ si )
-------- (9)

Taking the limit Δ si 0 for both sides of Eqn. (9) at back away point Eqn. (9) becomes;
lim
∆ si → 0 ( ∆∆ Ks )= dKds =0 ⋯ ⋯ (10)
i
−Q( s)
From Eqn. (3) K= P(s ) …………………… (11)

Differentiating Eqn. (11) w. r. t s, ds =


[ P (s) ]
2
dK
P ( s)
ds

−1
[ dQ ( s )
ds ]
Q ( s ) dP(s )
--------- (12)
From (10) and (12), at breakaway point,
52

Q ( s ) dP( s) P(s )dQ(s)


− =0 ------- (13)
ds ds

( P (s) )
Differentiating [G(s)H(s)] = differentiating K Q( s) w.r.t.s (i.e. from Eqn. 2)
d [ G ( s ) H (s ) ]
ds
=
k
[ Q(s)]
2 [
Q (s )
dP(s)
ds
−P( s)
dQ ( s)
ds
------ (14) ]
Substituting Eqn. (13) in (14) at breakaway point,
d [ G ( s ) H (s ) ]
=0 (Proved)
ds
Eq. 6: Determine the breakaway points of the root locus for loop transfer function
K
G ( s ) H ( s )=
( s+2 ) (s+ 4)
[G ( s ) H ( s ) ] K ( s +2 ) ( s+ 4 ) × 0−K (2 s+6)
d =d [ ]=
Solution: ds ( s +2 ) ( s+ 4 ) ( s2 +6 s+8 )
2

ds
Or 2s = -6 or s = -3 jω

B.P
σ
-4 -3 -2 -1

E.g. 7: Determine the breakaway points of the system which have the loop transfer function
-2
K ( s +4 ) K ( s+ 4 )
G ( s ) H ( s )= 2 = -
( s +2 s +4 ) ¿¿
2

d [ G ( s ) H (s ) ] ( s2 2 s+ 4 ) K −K ( s+ 4 ) (2 s+ 2)
= 2
=0 Or s2+8s+4=0
ds ( s2 +2 s +4 )

 s1 , s 2=−4 ± √ 12=−7.46∨−0.504
S-plane
Root Locus
X

- -
- -9- -5 -4 -1
8- 7- -6 -- -
10-- 9-- - -
-- - -6 3
-10 9 B.P -5 4 1
-8 7 X
E.g. 8: Determine Breakaway Points for
K K
G ( s ) H ( s )= =
s ( s +2 ) (s +2 s+ 10) s ( s +2 ) ( s+1+ j √3 ) (s+ 1− j √ 3)
2

d ¿¿
Or 4 s 3 +12 s2 +28 s+20=0 or 4 s 3 +4 s 2+8 s2 +8 s+20 s+20=0
Or (s+1)(s+1+2j)(s+1-2j)=0; Hence s = -1
= -1 -2j

= 1+ 2j jω

-
53

j3
j2 s-plane
j1

-6 -5 -4 -3 -2 -1 -j1
-j2
B. P -j3

The main disadvantage of this method – I is that for loop transfer function having higher
orders of s is cumbersome.
Procedure – II (for determining the breakaway points)
(1) Ch. Eqn. is 1+ F(s) = 1+G(s)H(s) aₒs n +a 1 sn−1 +… … … … . an =0
(2) Differentiate the Ch. Eqn. w. r. t. s and obtain:
F’(s) = bosn-1+ b1 sn-2 + …………… + bn-1 = 0
(3) Arrange the coefficients of s in expressions 1+F(s) and F’(s) as follows:
ao a1 a2 …………………………. an – 1 an
bo b1 b2 …………………………. bn – 1 o
(4) The tabulation is done in the following manner.
(a) The term sj (where j = 0, 1, 2……………n) of each row represents a particular row in the array.
(b) The term sj repeats for three consecutive rows for the value = n – 1… j.
(c) The first and third rows of the three rows which form the group sj in the array are identical.
(d) If the Ch. Eqn. 1 + F(s) contain multiple order roots, a row of the tabulation contains all
zeros, there by having pre-matured tabulation.
E.g. (Example): 1+ F ( s ) =ao s 3 +a1 s 2 +a2 s +a3 =0

Hence F ' ( s )=¿

s
3
a0 a1 a2 a3

s2 b0 b1 b2 0
bo a1 −b1 a0 bo a2 −b2 a0 bo a3 −0 ×c 0
s
2
b0
=c 0
b0
=c 1
b0
=c 2 0

s
2
b0 b1 b2 0
bo c 1−b 1 c 0 bo c 2−b 2 c 0 bo ×0−0 × c0
s
1
=d 0 =d 1 =d 2=0 0
d0 b0 b0

d o b1−d 1 b0 d o b2−d2 b0 d o × 0−0 ×b 0


s
1
=e0 =e1 =e2 =0 0
d0 d0 d0

s1 d0 d1 0 0
d o e1 −d 1 e 0 d o × 0−0 ×e 0
s
0
=f 0 =f 1=0 0 0
d0 d0
54

(5) Determine the breakaway point by solving the equation formed by the row of coefficient
which is just before the row of zeros.
E.g. 9: Determine the double order roots of the Eqn. s4 + 8 s 3+ 22 s2 +24 s+ 9=0

Solution: 1+ F ( s ) =s 4 +8 s3 +22 s 2+ 24 s +9=0= ( s+ 1 )2 ( s+3 )2


d ' 3 2
Hence ds ( 1+ F ( s ) )=F ( s ) =4 s +24 s + 44 s+24=0 or

s3 +6 s3 +11 s +6=0=F ' (s)

s
4
1 8 22 24 9

s3 1 6 11 06 0
1× 8−6 × 1 1× 22−11× 1 1× 24−6 ×1 1× 9−0× 1
s
3
1
=2
1
=11
1
=18
1
=9 0

s3 1 6 11 6 0
1× 11−6 ×2 1× 18−11 ×2 1× 9−6 × 2
s
2
=−1 =−4 =−3 0 0
1 1 1

−1× 6−−4 × 1 −1× 11−−3 ×1 −1× 6−0


s2 =2 =8 =6 0 0
−1 −1 −1

s
2
-1 -4 -3 0 0
… …( A . E)

−1× 8−−4 × 2 −1× 6−−3× 2


s
1
=0 =0 -3 0 0
−1 −1

The above array, the row of for s’ is zero. Hence break away points are obtained from the
Eqn. formed by the coefficients in the third row for s2 group is

-s2 -4s -3 =0 or s2 + 4s + 3 =0 or (s+3)(s+1) = 0  s = -1 and -3

E.g. 10: The open loop transfer function of a unity feedback control system is
K
G ( s )= . Determine the value of K and s at the breakaway point of the root locus.
( s +2 ) (s +3)

K 2
Solution: 1 + G(s) H(s) = 1+ ( s+ 2 )( s+3 ) = ( s+ 2 )( s+3 )+ K =0=s + 5 s+( K +6)

Hence F ' ( s )=2 s+5

s
2
1 5 (K+6)

s
1
2 5 0
55

2× 5−5 ×1 2×(K +6)−0 ×1


s
1
2
=2.5 =( K + 6) 0
2

s
1
2 5 0
2 ( K +6 )−5 ×3.5
s
0
0 0
2

In the above array the row for s0 is not zero. To complete the array this row should have
2 ( K +6 )−5 ×2.5
zero coefficients that is 2
=0 or 2K+12-12.5 = 0 or K = 0.25 .

Hence the Auxiliary Eqn. is 2s+5=0→ s= -2.5 is the breakaway point at which there is
multiple roots i.e. Ch. Eqn. Is s²+5s+ (K+6) = 0 or s²+5s+6.25 = 0 or (s+2.5)² = 0 indicates
double roots at s = -2.5

E.g. 11: Calculate the values of K and the location of breakaway points of the system
K (s+3)
defined by the loop transfer function: G ( s ) H ( s )= s(s+2)
Solution: 1+G(s) H (s) = s(s+2) +K(s+3) =0=s2+s (K+2) +3K=0
Hence F’(s) =2s+ (K+2) =0
s
2
1 (K+2) 3K
s
1
2 (K+2) 0
2 ( K +2 ) −( K +2 ) 1 K +2 2× 3 K −0 ×1
s1 =¿
2 2
=3K 0
2
s
1
2 (K+2) 0
2× 3 K −( K +2 ) ( K +2)/2
s
0
0 0
( K +2)/2
Hence 2x3K – (K+2)2/2=0 or 12K – K2-4K-4 = 0
8 ± √ 64−16
Or K2 – 8K + 4 = 0 or K= =4 ± √ 16−4=4 ±2 √ 3= 7.4641, + 0.535
2
The Auxiliary Equation is 2s+ (K+2) =0
Hence s = -4.74 for K=7.48
= -1.26 for K=0.5
(11) Value of K at a point on the Root Locus
The value of K at a point s1 on the root locus is determined by measuring the vectors
from the poles and zeros of loop transfer function to the point’s s 1 on the root locus and
then substituting the values in the following:
n+ m
∏ |s1 + p j|
1 j=i ¿
K= = =Product of length of vectors ¿ the poles of G ( s ) H (s)¿ s 1
|G ( s ) H ( s )| n Product of length of vectors ¿
∏ |s 1+ z i̇|
i=1

Root Locus Jω
BDE
S1 K=
AC

s-Plane
E
56

B C D

Procedure for construction of Root Locus


(1) Starting point of root locus : Location of finite poles = p
(2) Terminating points of root locus: Location of finite zeros = z
(3) Number of separate root loci : N = P or z whichever is greater
(4) Symmetry of root locus : Root locus plots are symmetrical about real
axis
(5) Number of asymptotes : P – Z =m
( 2 k +1 ) π
(6) Angel of asymptotes for the root locus : θ= P−Z for k=0, 1 ----------- {(p-z) exclusive}
At s  ∞
(7) Centroid of the intersection point : σ 1=
∑ poles of G ( S ) H ( S )−∑ zeros of G ( s ) H (s )
p−z
Of asymptotes
(8) Section of Root Locus on the real axis: Count the odd number of real poles and zeros so
that the left of these real poles and zeros, root locus will lie on the real axis.

(9) A angle of departure and arrival at the:θd = ―[(2k+1) π+(θ p +θ p + …+θ p ) ―( 1 2 m +n

θ z +θ z + …+θ z )] 1 2 n

θ a= [(2k+1) π+ (θ p +θ p + …+θ p )
1 2 m +n ―(
θ z +θ z + …+θ z )]
1 2 n

(10) Point of intersection of root locus with: Use the Routh Array
the imaginary axis in s-plane.
d [G ( s ) H ( s ) ]
(11) Breakaway or Saddle Points on the real axis : (i) =0 (ii) Similar to Routh
ds
Array
(12) Value of K on the root locus at point s1:
1 ¿
K= =Product of all vector length ¿ poles of G ( s ) H ( s ) ¿ s 1 zeros of G ( s ) H ( s ) ¿
|G ( s ) H (s )| Product of all vector length ¿

K
E.g. 12: Sketch the root locus of the unity feedback system having G ( s )= S ( s+ 4 ) (s +2) when K
is varied from 0 to ∞. Hence obtain the value of K for which the system is unstable.
(1) P = 3, Z = 0, p1= 0, p2 = -2, p3 = -4, N = P =3
[ 0−2−4 ]−0 π
(2) M = P – Z = 3 – 0 = 3 ; σ 1= =−2 , θ1= =60 ° for k =0
3
3
(2 k +1)π 3π
θ= for k =0,1,2 … . θ2 = =180 ° for k =1
p−z 3

θ3 = =300 ° for k =2
3
57

There is no complex pole or zero so θ d and θ a are not required.

K
1+ =0 or s 3 + 6s 2 +8s +K=0=ch. Eqn.
s ( s+2 ) ( s+ 4)

Routh Array: S 3 1 8

S² 6 K  A.E is 6s2 + k = 0 or s = √ 8 for k = 48


6 ×8−K
S1 6
=0 0 48 – K =0  K = 48
S0 K 0

Hence the intersection point of root locus with Imaginary Axis is at s = ±2.822j for K = 48

(3) Breakaway points: 1+F(s) = Ch. Eqn. = s3 + 6s2 +8s + K = 0


Hence F’(s) = 3s2 -++12s + 8 = 0
The Array: s3 1 6 8 K
2
S 3 12 8 0
3× 6−12× 1 3× 8−8 ×1 16 3 K −0 ×1
S2 3
=2
3
=
3 3
=K 0
S2 3 12 8 0
16
1 3× −12× 2 3 K −8 ×2 3 K−16
S 3
=
−8
3
=
3
0 0 …….A.E
3 3
−8 3 K−16
×12− ×3 3(3 K +16)
S1 3 3 ¿ 0 0
8
−8/ 3
3 K −16
S1 -8/3 3
0 0
−8 3 K −16 3 (3 K +16)
×8− ×
S0 3 3 8
⇒0
0 0
−8/3
=27K² – 768 + 512  K = ± 3.08. And Auxiliary Equation obtained corresponding
−8 3 K−16
to s1 row is as: 3 s+ 3 =0 ; s=-0.845 for K = 3.08

3j s-plss-
plane
+2.822j (for K =48)
σ 1=−2 2j
1j
-6 -5  -3 -2
-4  -1 𝜎
∞ K -1j
K -2.822j

Fig . G ( s ) H ( s )= ¿
s ( s+ 2 ) s+ 4 ¿ -3j
1 -0.845
=B.P

E.g. 13: Sketch the root locus of the rational loop transfer function G(s)¿
58

(i) P=2,Z = 1, N = P = 2 since P> Z


(ii) No. of asymptotes = m = P – z = 2 – 1 = 1
( 2 k +1 ) π
(iii) Angle of Asymptotes ¿ θ= P−Z =180° for k=0
[ −3−2 j−3+2 j ]−(−4)
(iv) б 1 = =−2
( P−Z )
(v) Angle of departure: θd =−[ 180+θ p −θ z ]=− [180+ 90−tan 2 ]=206.6 °
−1
1 2 1

(vi) Point of intersection with Imaginary Axis: Ch. Eqn. is θd2 = +206.6°
K (s+ 4) 2
1+ 2
=0=s + ( K + 6 ) s +(4 K +13)
s + 6 s+ 13
Routh Array:
S2 1 (4K+13)

S1 (K+6) 0

( K +6 ) ( 4 K +13 )−0
S⁰ =(4 K + 13) 0
(K +6)

−13
Hence (4K+13) = 0 (limiting value) or K= 4 → for +ve value of K, the root locus will not
intersect the imaginary axis.

(vii) Breakaway points:


1+F(s) = s2+ (K+6) s = (4K+13) =0
Hence F’(s) =2s+ (K+6) =0
Hence S2 1 (K+6) (4K+3)

S1 2 (K+6) 0

2 ( K +6 )−( K +6 ) × i K +6 2(4 K + 13)


S1 = =( 4 K +13 ) 0 (A.E)
2 2 2

S1 2 (K+6) 0

( K +6)
0 2 ( 4 K +13 )− ( K + 6 )
S 2
=0
0 0
2

(k +6)
2 ( 4 k +13 )−( k +6 )
Hence 2
=0
or 4(4k+13) – (k+6)2=0
2
Or K=2± √ 20=6.64∨−2.46
−K +6
A.E : 2 s + ( K +6 ) =0 at K =6.64 , s= 2
=−6.32
59


+3j s-plane

+2j
-6.32 -206.6° +1j
-8 -7 -6 -5 -4 -3 -2 -1 -3j σ

B.P -2j
-153.4° -1j
E.g. 14: A system represented by the Fig. shown below: =+ 206.6°

Hence determine:

(a) Value of K which causes instability in system.


(b) For damping ratio 0.34 determine the value of K and the Gain margin
K
G ( s ) H ( s )=
s ( s +1 ) (s +4 )

(i) Z=0, P=3, N=P=3 since P>Z, p1 = 0, p2 = -1, p3= -4


(ii) m=P-Z = 3 – 0 = 3
[ 0−1−4 ] −0 −5
(iii) б 1 = = =−1.666 jω
3 3
(iv) θ1=⊼/3=60 ° for k =0
3⊼ 3j
θ 2= =180° for k =1
3
5⊼
θ3 = =300° for k =2 300° A 2j
3
K 180° 60° 1j
(v) Ch. Eqn. 1+ s ( s+ 4 ) (s+ 1) =0=s ³+5 s ²+ 4 s+ K
-4 -3 -2 -1 0 б
3
Routh Array: s 1 4 -1j
s2 5 KA.E -2j
5× 4−K ×1 20−K
s1 5
=
5
0 BP= -0.467 -3j
0
s K 0
20−K
Hence 5 =0→ K=20
A.E: 5s2 + K = 0 or s = ±2j for K = 20
d [ G ( s ) H ( s) ]
(vi) Breakaway point: =0=3 s 2 +10 s+ 4=0
ds
2
−K [3 s +10 s+ 4]
¿
2 2
Or s=−0.467 ;−2.86= −10 ± √ 100−48
( s +5 s 4 s )
3 2
6
60

(a) For values of K greater than 20, the Ch. Eqn. roots lie in the right side of Imaginary
axis of the s-plane there by making the system unstable.
(b) For the damping ratio δ=0.39 θ=cos-1 0.34 = 70°
The point A at which the root locus intersection line is defined by s=-0.34 + 0.94j in
the space, at the value of K are =K1: i.e.
1
K 1= =( 0.34+ 0.94 j ) ( 0.66+0.94 j )( 3.66+ 0.94 )=4.03
|G ( s1 ) H ( s 1)|
(s1+p1) (s1+p2) (s1+p3)
Thus for δ=0.34, the value of K=4.03
4.03
For K=4.03 the laptop transfer function changes to G(s)H ( s)= s ( s+1 ) ( s+ 4)
The gain margin defined as = value of K at the Imaginary Axis/Design value of K= 20/4.03
2. Other Properties of Root Locus
(i) Addition of open loop poles: The addition of a pole to the system loop transfer
function pulls the root locus to the right of s-plane (i.e. close to the Imaginary
Axis) thereby making the system less stable and slow in response.
k K k
E.g. G ( s ) H (s) s( s+ a) −→ s ( s+ a ) (s+ b) → s ( s+ a ) ( s +b ) (s+ c)



S-plane s-plane
jω S-plane
K=0 B4
-a o б
-b -a o б -c -b -a B3 o б
B1
B1 B2 B1 B2

K K K
Fig: (a): G ( s ) H ( s )= s( s+ a) Fig. (b): G ( s ) H ( s )= s ( s +a ) (s+ b) Fig: (c):G ( s ) H ( s )= s ( s +a ) ( s +b )( s+ c)

(ii) Addition of zeros: The addition of zero to the system produces the effect of pulling
the root locus to the left (i.e. away from the Imaginary Axis) of s-plane, thereby
making the system to be more stable and faster in response.
K (s+ b) K (s 2 +cs +d)
E. g. G ( s ) H ( s )= s(s+ a) s (s+a)

B’2 B2 Jω
Jω Axis B’2 B2
s-plane O
-b -a o
o б O o
-a -b -a
B1
K K (s+ b)
Fig . ( a ) :G ( s ) H ( s )= Fig. (b): G ( s ) H ( s )=
s ( s+a) s( s+ a)

S-plane
B3
б
B2 61
B1

-a 0

2
K (s +cs +d )
Fig: (c): G ( s ) H ( s )=
s( s+ a)

3. Root Contours

The loci of system characteristic roots for different parameter (other than K) variations are
selected by using the generalized roots locus procedure. These loci are known as Root
Contours of the system.
KP (s )
E.g. G ( s ) H ( s )= Q (s) ; Hence Ch. Eqn. is 1+G(s) H (s) =Q(s)+KP(s)=0……… (I)
Eqn. (I) contains a variable parameter T other than K in polynomials P(s) & Q(s) which is
varied from -∞ to +∞ keeping K constant, hence Eqn. (I) becomes, Q(s)+KP(s)=Q 1(s)+T
P₁(s)=0 …… (2)
The locus of the roots of Eqn. (2) when T is changed from - ∞ to +∞ with K as
constant is the Root Contour of the system and is obtained from the pole & zero
configuration P(s)/Q(s) by applying the procedure described in the rules of construction of
root locus.
E. g: R(s) + C(s)

-
Sketch the root contour of the system when
parameter x is changed from - ∞ to + ∞.
K
4 (s+10) 4 ( s+10)
Solution: G ( s ) H ( s )= s (s + x) ; 1+G ( s ) H ( s )=1+
s (s + x)
=0

Or s (s + x) + 4(s+10) = 0 or (s2+4s+40) + x s=0; divided by s2+4s+40=0 in the above equation.


xs
We get, 1+G₁ (s)H₁ (s)=1+ 2
=0 (so the same rules for construction of root locus shall
s + 4 s+ 40
be applicable to this equation)
x P1 (s ) xs xs
Hence G1 ( s ) H 1 ( s )= Q ( s) = 2 =
1 s + 4 s +40 ( s+2+ j 6 ) (s +2− j 6)
(i) Poles at s = -2 –j6; s = -2+j6  The root contour starts from these points and P=-2+j6
(ii) A zero at s=0 and Z=1
(iii) N = No. of separate root contour = P~Z=2
(iv) For +ve value of x the asymptotes m=P-Z=2-1=1 and the angle of asymptote
π (2 k +1)π
θ 1= = =180 ° (for k=0)
1 p−z
[ −2−6 j−2+ 6 j ] −0=−4
(v) б 1 =centroid= (2−1)
(vi) The section of root contour on the real axis:
62

(vii) Intersection of root contour with the Imaginary axis: Ch. Eqn. is s2+4s+40+xs=0
Routh array: s2 1 40
s¹ 4+x 0
( 4+ x ) 40
s° =40 0
(4 + x)
Auxiliary eqn. is obtained corresponding to the row of s2 and 4+x=0 x= - 4
AE is s²+40 or s= ± j 6.32
(viii) θd 1=−[ 180 °+ 90° −180 ° ] =−162°
θd 2=−[ 180 °+ 270° −252° ] =−198 °
(ix) Breakaway points: 1+F1 (s) = s2 + (x+4)s + 40 = 0
Hence s² 1 (x+4) 40
s¹ 2 (x+4) 0
2 ( x +4 )−(x+ 4) (x + 4) 2× 40−0
s¹ =
2
=40 0
2 2
s¹ 2 (x+4) 0……A.E
( x +4 )∗( x +4)
2× 40−
s° 2 0 0
2
( x + 4 )2
Hence 80− =¿ x=6.7∧−14.7
2
−( x+ 4)
A.E is 2s+x+4=0 or s=
2
Hence at the breakaway points: s= -5.35 for x= 6.7
S= 5.35 for x =-14.7


+6j
P1 S-plane

∞X
Z б

S=-5.35 S=5.35
x=6.7 X = -14.7

P2
-6j
4. System with Transportation Lag
All the feedback systems discussed so far is represented by linear, lumped parameter
mathematical models (transfer functions consisting of the ratios of algebraic
polynomials). This is valid so long as the time taken for energy transmission is negligible
i.e. the output begins to appear immediately on application of input. This is not quite
true of transmission channel-lines, pipes belt conveyors, etc. In such cases a definite time
63

elapses after application of the input before the output begins to appear. This type of
pure time lag is known as transportation lag or dead time.
Linear lumped parameter model is valid under such situations unless the pure time
log is negligible compared to other logs in the system. For example, the transmission line
(pipe) between the hydraulic pump and motor causes a time lag in transportation of oil
from pump to motor and vice versa. Similarly the slow rate of transmission of heat
energy by conduction or convection introduces serious transportation logs in process
control systems.
The transfer function corresponding to a transportation lag can be easily determined.
Consider a component whose output is the same as the input but delayed by T sec.
Mathematically this can be expressed as C o(t) = C i (t – T)
Taking Laplace transform of the above equations, we get,
Co (s) − ST
Co(s) = C i (s) e –s t or C ( s) =e = Transfer function for a constant transportation lag. Thus
i

the transfer function for a constant transportation lag is given by e –s T


C a non – minimum phase function: the transfer function has one or more zeros in the
right half s-plane, the phase angle varies from 0° to 270° or more. But minimum phase
function has all poles and zeros in the left half side of s-plane and the phase angle varies
from 0 to -90°. And all pass system transfer function has anti symmetry pole – zero
patterns and the phase angle varies from 0 to -180° (Nagrath page 265 & 266)
E.g. the system represented below shows arrangements of controlling the thickness of a
still plate produced by rolling mill. A voltage signal corresponding to the desired
thickness is the reference (Which has been calibrated/decided experimentally/
analytically) input to the system. A thickness gauge provides a feedback voltage signal,
proportional to the actual thickness of the plate. The error voltage actuates the motor
which positions the rolls.

Reference Voltage +
Amplifier Motor
(Signal corresponding to
desired thickness) Feedback voltage
(Signal proportional to Gear
The actual thickness)
Gauge Controlled Roll

Fixed roll
Fig: A feedback system with transportation lags.

1 K
The open loop transfer function of the system is given by G(s) H (s)= s(τ s +1) where for
m

simple analysis; we assume a single dominant time constant in the loop, i.e. probably in
64

the motor. In the above transfer function, we have ignored the fact that a finite time
must elapse before a change in thickness of still plates between rolls reaches the point of
measurement at the gauge. If this delay in transportation of signal is assumed to be T
K 1 e−sT
sec, then the open loop transfer function of the system is G( s ) H ( s )= .
s (τ m s +1)
K e−sT
The above equation may be rearranged in the following form: G( s) H ( s )=
s( s+ L)
Putting the root locus for a system with transportation lab is considerably more
complicated then the system without it. However if the transportation log is small

e−sT
≃(1−sT ), then we can write,
G ( s ) H ( s )=
− s− ( T1 )
s ( s+ α )

The Ch. Eqn. becomes 1+G ( s ) H ( s )=1−


K s−
T
1
(
=1−P ( s )=0
)
where K=
K1
τm
comparing the ( )
s ( s +α )
above equation with the Eqn. [1+P(s)=G(s)H(s)]=0. We observe that the angle criterion
i.e. is modified as:

(Such cases also occur in positive feedback system)


This leads to following modification in the rules of constructing the Root Locus:
(i) A point on the real axis lies on the root locus if the number of finite real poles plus
real zeros to the right of this point is Even instead of odd.
(ii) In the expression of Angle of Asymptote, Angle of departure and Angle of Arrival is
180(2k) in stead 180(2k+1)
Example: α =2 , T =1 sec
Where e−st is appromimated as (1−sT )
−K (s−T )
G(s) H (s)= jω
s ( s+2)
J√2 (K=2)

1 б
-3 -2 -1

-J√2 (K=2)

[
Fig: Root locus plot of 1+
Ke −sT

s( s+ 2)
=0 ] K=2

5. Sensitivity of the Roots of the Ch. Eqn.:


The sensitivity of the transfer function M(s) to variation of a parameter is defined as:
d (Ln M ) ∂ M / M
s MK = = ……………….. (1)
d ( Ln K ) ∂ K / K
65

G
Where, M = 1+GH and K is the parameter of interest. It is useful to define the sensitivity
of the roots of the ch.eqn. (i. e. the closed loop poles of the systems) to variation of a
parameter K i.e.
∂(−rK ) ∂(−rK )
−r K
sK = = ……………… (2)
∂( K) ∂K/K
Where –r K is a root of the Ch. Eqn.
In order that the root of the system dynamic response be relatively unaffected by
parameter changes, the system must be so designed that the roots of the Ch. Eqn. in
particular the dominant roots are made insensitive to parameter changes i.e. the roots
sensitivity s−rk
k is made less than a specified value.
K (s + z)
E.g. Given Ch. Eqn. is1+ s (s + p) =0 , the parameter of interests are the gain K, the zero is at
s = -z and the pole is at s = -p
(Unwanted change is any of these parameters will cause the roots of the Ch. Eqn. to
shift. The amount of nature of nature of this shift is determined by the root sensitivity to
each of these parameters. Let us consider the root sensitivity to these parameters one by
one).
(a) Root sensitivity to gain K
E.g. Given two open – loop poles are at s = 0, s = -2, and the zero at s = -3.
K (s +3)
Hence Ch. Eqn. becomes 1+ s (s +2) =0
It is seen that part of the root locus is a circle with centre at s = -3
Let the nominal gain the system be K = K0 = 4 given.
Ch. Eqn. is s(s+2) + K(s+3) = s2+2s+4s+12=0 or s2 +6s + 12 = 0
−6 ± √ 36−48
S1, S2 = =−3 ± √ 9−12=−3 ± j √3
2

r1=-3 + j √3
S=-3.3 + j 1.7 S=-2.7 + j 1.7 jω

j 1.7

K=∞
-∞ <- K
б
-5 -4 -3 -2 -1

j 1.7
K=484 K=3.44

This results in two complex roots,


s1 = -r1 = -3+ j √3 and s2 = -r2 = -3 -j√3 = r1*
66

Since the roots are complex conjugate, the root sensitivity for –r, is the conjugate of
the root sensitivity for –r2. Therefore we need to evaluate only the root sensitivity of
one of the roots, say –r1
Assume that because of changes in gain K, the root –r 1 gets shifted to a new location s
= -3.3 +j 1.7 on the root locus of course.
Then Δ (-r1) = (-3.3 + j 1.732) – (-3+J1.732) = -0.3 – j 0.32
From the root locus plotted above, the value of gain K for this new location of the
root (obtained by the magnitude criterion) is 4.84
Therefore ΔK = K – K0 = 4.84 – 4.0 = 0.84 hence
∆ K 0.84
Hence K = 4 =0.21
s
The sensitivity of the root –r1 is: K+ ¿ (−r 1)=
∆ (−r1) −0.3− j 0.032
∆ K/K
=
0.21
=1.43 186° ¿ for the changes of gain.
Considering again the new root location as s = -2.7 + j1.7, the gain corresponding to
this location obtained from the root locus drawn above is 3.44
Therefore Δ (-r1) = (-2.7 + j 1.7) – (3 + j 1.732) = (0.3 – j 0.032)
∆K 0.56
ΔK = 4-3.44 = 0.56 and hence K = 4 =0.14
The sensitivity of the root –r1 is now given by
(0.3− j 0.03)
s(−r
K =
1 )
=2.16−6 ° For negative change of gain:
0.14
∆K
It can easily be verified that the percentage change in gain K decreases the
sensitivity measures s K+ ¿(−r )¿ and s K−¿(−r )¿ approach equality in magnitude and a
1 1

∆K
difference in angle of 180°. Thus for small changes in gain K the sensitivity
measures are related as:
(−r ) (−r )
¿ …………. (3); ∠ sk +¿ =180 ° +∠ sk +¿ ¿ ¿…………… (4)
1 1

Usually the desired root sensitivity measure is for small changes in the parameter.
Therefore we need to evaluate only one sensitivity measure, say for positive changes
in parameters; other sensitivity measure (for negative change of parameters) can be
obtained by use of equations (3) and (4).

You might also like